These material are compiled for helping junior / senior software engineers and others.

1. What is Log Switch ?

The point at which ORACLE ends writing to one online redo log file and begins writing to another is called a log switch.

2. What is On-line Redo Log?

The On-line Redo Log is a set of tow or more on-line redo files that record all committed changes made to the database. Whenever a transaction is committed, the corresponding redo entries temporarily stores in redo log buffers of the SGA are written to an on-line redo log file by the background process LGWR. The on-line redo log files are used in cyclical fashion.

3. Which parameter specified in the DEFAULT STORAGE clause of CREATE TABLESPACE cannot be altered after creating the tablespace?

All the default storage parameters defined for the tablespace can be changed using the ALTER TABLESPACE command. When objects are created their INITIAL and MINEXTENS values cannot be changed.

4. What are the steps involved in Database Startup ?

Start an instance, Mount the Database and Open the Database.

5. What are the steps involved in Instance Recovery ?

Rolling forward to recover data that has not been recorded in data files, yet has been recorded in the on-line redo log, including the contents of rollback segments. Rolling back transactions that have been explicitly rolled back or have not been committed as indicated by the rollback segments regenerated in step a. Releasing any resources (locks) held by transactions in process at the time of the failure. Resolving any pending distributed transactions undergoing a two-phase commit at the time of the instance failure.

6. Can Full Backup be performed when the database is open ?

No.

7. What are the different modes of mounting a Database with the Parallel Server ?

Exclusive Mode If the first instance that mounts a database does so in exclusive mode, only that Instance can mount the database.
Parallel Mode If the first instance that mounts a database is started in parallel mode, other instances that are started in parallel mode can also mount the database.

What are the advantages of operating a database in ARCHIVELOG mode over operating it in NO ARCHIVELOG mode ? Complete database recovery from disk failure is possible only in ARCHIVELOG mode. Online database backup is possible only in ARCHIVELOG mode.

8. What are the steps involved in Database Shutdown ?

Close the Database, Dismount the Database and Shutdown the Instance.

9. What is Archived Redo Log ?

Archived Redo Log consists of Redo Log files that have archived before being reused.

10. What is Restricted Mode of Instance Startup ?

An instance can be started in (or later altered to be in) restricted mode so that when the database is open connections are limited only to those whose user accounts have been granted the RESTRICTED SESSION system privilege.

11. What is Partial Backup ?

A Partial Backup is any operating system backup short of a full backup, taken while the database is open or shut down.

12. What is Mirrored on-line Redo Log ?

A mirrored on-line redo log consists of copies of on-line redo log files physically located on separate disks, changes made to one member of the group are made to all members.

13. What is Full Backup ?

A full backup is an operating system backup of all data files, on-line redo log files and control file that constitute ORACLE database and the parameter.

14. Can a View based on another View ?

Yes.

15. Can a Tablespace hold objects from different Schemes ?

Yes.

16. Can objects of the same Schema reside in different tablespaces.?

Yes.

17. What is the use of Control File ?

When an instance of an ORACLE database is started, its control file is used to identify the database and redo log files that must be opened for database operation to proceed. It is also used in database recovery.

18. Do View contain Data ?

Views do not contain or store data.

19. What are the Referential actions supported by FOREIGN KEY integrity constraint ?

UPDATE and DELETE Restrict - A referential integrity rule that disallows the update or deletion of referenced data. DELETE Cascade - When a referenced row is deleted all associated dependent rows are deleted.

20. What are the type of Synonyms?

There are two types of Synonyms Private and Public.

21. What is a Redo Log ?

The set of Redo Log files YSDATE,UID,USER or USERENV SQL functions, or the pseudo columns LEVEL or ROWNUM.

22. What is an Index Segment ?

Each Index has an Index segment that stores all of its data.

23. Explain the relationship among Database, Tablespace and Data file.?

Each databases logically divided into one or more tablespaces one or more data files are explicitly created for each tablespace

24. What are the different type of Segments ?

Data Segment, Index Segment, Rollback Segment and Temporary Segment.

25. What are Clusters ?

Clusters are groups of one or more tables physically stores together to share common columns and are often used together.

26. What is an Integrity Constrains ?

An integrity constraint is a declarative way to define a business rule for a column of a table.

27. What is an Index ?

An Index is an optional structure associated with a table to have direct access to rows, which can be created to increase the performance of data retrieval. Index can be created on one or more columns of a table.

28. What is an Extent ?

An Extent is a specific number of contiguous data blocks, obtained in a single allocation, and used to store a specific type of information.

29. What is a View ?

A view is a virtual table. Every view has a Query attached to it. (The Query is a SELECT statement that identifies the columns and rows of the table(s) the view uses.)

30. What is Table ?

A table is the basic unit of data storage in an ORACLE database. The tables of a database hold all of the user accessible data. Table data is stored in rows and columns.

31. What is a Synonym ?

A synonym is an alias for a table, view, sequence or program unit.

32. What is a Sequence ?

A sequence generates a serial list of unique numbers for numerical columns of a database's tables.

33. What is a Segment ?

A segment is a set of extents allocated for a certain logical structure.

34. What is schema?

A schema is collection of database objects of a User.

35. Describe Referential Integrity ?

A rule defined on a column (or set of columns) in one table that allows the insert or update of a row only if the value for the column or set of columns (the dependent value) matches a value in a column of a related table (the referenced value). It also specifies the type of data manipulation allowed on referenced data and the action to be performed on dependent data as a result of any action on referenced data.

36. What is Hash Cluster ?

A row is stored in a hash cluster based on the result of applying a hash function to the row's cluster key value. All rows with the same hash key value are stores together on disk.

37. What is a Private Synonyms ?

A Private Synonyms can be accessed only by the owner.

40. What is Database Link ?

A database link is a named object that describes a "path" from one database to another.

41. What is a Tablespace?

A database is divided into Logical Storage Unit called tablespaces. A tablespace is used to grouped related logical structures together

42. What is Rollback Segment ?

A Database contains one or more Rollback Segments to temporarily store "undo" information.

43. What are the Characteristics of Data Files ?

A data file can be associated with only one database. Once created a data file can't change size. One or more data files form a logical unit of database storage called a tablespace.

44. How to define Data Block size ?

A data block size is specified for each ORACLE database when the database is created. A database users and allocated free database space in ORACLE datablocks. Block size is specified in INIT.ORA file and cant be changed latter.

45. What does a Control file Contain ?

A Control file records the physical structure of the database. It contains the following information.
Database Name
Names and locations of a database's files and redolog files.
Time stamp of database creation.

46.What is difference between UNIQUE constraint and PRIMARY KEY constraint ?

A column defined as UNIQUE can contain Nulls while a column defined as PRIMARY KEY can't contain Nulls.

47.What is Index Cluster ?

A Cluster with an index on the Cluster Key

48.When does a Transaction end ?

When it is committed or Rollbacked.

49. What is the effect of setting the value "ALL_ROWS" for OPTIMIZER_GOAL parameter of the ALTER SESSION command ? What are the factors that affect OPTIMIZER in choosing an Optimization approach ?

Answer The OPTIMIZER_MODE initialization parameter Statistics in the Data Dictionary the OPTIMIZER_GOAL parameter of the ALTER SESSION command hints in the statement.

50. What is the effect of setting the value "CHOOSE" for OPTIMIZER_GOAL, parameter of the ALTER SESSION Command ?

The Optimizer chooses Cost_based approach and optimizes with the goal of best throughput if statistics for atleast one of the tables accessed by the SQL statement exist in the data dictionary. Otherwise the OPTIMIZER chooses RULE_based approach.

51. What is the function of Optimizer ?

The goal of the optimizer is to choose the most efficient way to execute a SQL statement.

52. What is Execution Plan ?

The combinations of the steps the optimizer chooses to execute a statement is called an execution plan.

53. What are the different approaches used by Optimizer in choosing an execution plan ?

Rule-based and Cost-based.

54. What does ROLLBACK do ?

ROLLBACK retracts any of the changes resulting from the SQL statements in the transaction.

55. What is SAVE POINT ?

For long transactions that contain many SQL statements, intermediate markers or savepoints can be declared which can be used to divide a transaction into smaller parts. This allows the option of later rolling back all work performed from the current point in the transaction to a declared savepoint within the transaction.

56. What are the values that can be specified for OPTIMIZER MODE Parameter ?

COST and RULE.

57. What is COST-based approach to optimization ?

Considering available access paths and determining the most efficient execution plan based on statistics in the data dictionary for the tables accessed by the statement and their associated clusters and indexes.

58. What does COMMIT do ?

COMMIT makes permanent the changes resulting from all SQL statements in the transaction. The changes made by the SQL statements of a transaction become visible to other user sessions transactions that start only after transaction is committed.

59. What is RULE-based approach to optimization ?

Choosing an executing planbased on the access paths available and the ranks of these access paths.

60. What are the values that can be specified for OPTIMIZER_GOAL parameter of the ALTER SESSION Command ?

CHOOSE,ALL_ROWS,FIRST_ROWS and RULE.

61. Define Transaction ?

A Transaction is a logical unit of work that comprises one or more SQL statements executed by a single user.

62. What is Read-Only Transaction ?

A Read-Only transaction ensures that the results of each query executed in the transaction are consistant with respect to the same point in time.

63. What is a deadlock ? Explain .

Two processes wating to update the rows of a table which are locked by the other process then deadlock arises. In a database environment this will often happen because of not issuing proper row lock commands. Poor design of front-end application may cause this situation and the performance of server will reduce drastically.
These locks will be released automatically when a commit/rollback operation performed or any one of this processes being killed externally.

64. What is a Schema ?

The set of objects owned by user account is called the schema.

65. What is a cluster Key ?

The related columns of the tables are called the cluster key. The cluster key is indexed using a cluster index and its value is stored only once for multiple tables in the cluster.

66. What is Parallel Server ?

Multiple instances accessing the same database (Only In Multi-CPU environments)

67. What are the basic element of Base configuration of an oracle Database ?

It consists of
one or more data files.
one or more control files.
two or more redo log files.
The Database contains
multiple users/schemas
one or more rollback segments
one or more tablespaces
Data dictionary tables
User objects (table,indexes,views etc.,)
The server that access the database consists of
SGA (Database buffer, Dictionary Cache Buffers, Redo log buffers, Shared SQL pool)
SMON (System MONito)
PMON (Process MONitor)
LGWR (LoG Write)
DBWR (Data Base Write)
ARCH (ARCHiver)
CKPT (Check Point)
RECO
Dispatcher
User Process with associated PGS

68. What is clusters ?

Group of tables physically stored together because they share common columns and are often used together is called Cluster.

69. What is an Index ? How it is implemented in Oracle Database ?

An index is a database structure used by the server to have direct access of a row in a table. An index is automatically created when a unique of primary key constraint clause is specified in create table comman (Ver 7.0)

70. What is a Database instance ? Explain

A database instance (Server) is a set of memory structure and background processes that access a set of database files.
The process can be shared by all users. The memory structure that are used to store most queried data from database. This helps up to improve database performance by decreasing the amount of I/O performed against data file.

71. WWhat is the use of ANALYZE command ?

To perform one of these function on an index,table, or cluster:
- To collect statistics about object used by the optimizer and store them in the data dictionary.
- To delete statistics about the object used by object from the data dictionary.
- To validate the structure of the object.
- To identify migrated and chained rows of the table or cluster.

72. What is default tablespace ?

The Tablespace to contain schema objects created without specifying a tablespace name.

73. What are the system resources that can be controlled through Profile ?

The number of concurrent sessions the user can establish the CPU processing time available to the user's session the CPU processing time available to a single call to ORACLE made by a SQL statement the amount of logical I/O available to the user's session the amout of logical I/O available to a single call to ORACLE made by a SQL statement the allowed amount of idle time for the user's session the allowed amount of connect time for the user's session.

74. What is Tablespace Quota ?

The collective amount of disk space available to the objects in a schema on a particular tablespace.

76. What are the different Levels of Auditing ?

Statement Auditing, Privilege Auditing and Object Auditing.

77. What is Statement Auditing ?

Statement auditing is the auditing of the powerful system privileges without regard to specifically named objects.

78. What are the database administrators utilities avaliable ?

SQL * DBA - This allows DBA to monitor and control an ORACLE database. SQL * Loader - It loads data from standard operating system files (Flat files) into ORACLE database tables. Export (EXP) and Import (imp) utilities allow you to move existing data in ORACLE format to and from ORACLE database.

79. How can you enable automatic archiving ?

Shut the database
Backup the database
Modify/Include LOG_ARCHIVE_START_TRUE in init.ora file.
Start up the database.

80. What are roles? How can we implement roles ?

Roles are the easiest way to grant and manage common privileges needed by different groups of database users. Creating roles and assigning provides to roles. Assign each role to group of users. This will simplify the job of assigning privileges to individual users.

81. What are Roles ?

Roles are named groups of related privileges that are granted to users or other roles.

82. What are the use of Roles ?

REDUCED GRANTING OF PRIVILEGES - Rather than explicitly granting the same set of privileges to many users a database administrator can grant the privileges for a group of related users granted to a role and then grant only the role to each member of the group.
DYNAMIC PRIVILEGE MANAGEMENT - When the privileges of a group must change, only the privileges of the role need to be modified. The security domains of all users granted the group's role automatically reflect the changes made to the role.
SELECTIVE AVAILABILITY OF PRIVILEGES - The roles granted to a user can be selectively enable (available for use) or disabled (not available for use). This allows specific control of a user's privileges in any given situation.
APPLICATION AWARENESS - A database application can be designed to automatically enable and disable selective roles when a user attempts to use the application.

83. What is Privilege Auditing ?

Privilege auditing is the auditing of the use of powerful system privileges without regard to specifically named objects.

84. What is Object Auditing ?

Object auditing is the auditing of accesses to specific schema objects without regard to user.

85. What is Auditing ?

Monitoring of user access to aid in the investigation of database use.

86. What are the responsibilities of a Database Administrator ?

Installing and upgrading the Oracle Server and application tools. Allocating system storage and planning future storage requirements for the database system. Managing primary database structures (tablespaces) Managing primary objects (table,views,indexes) Enrolling users and maintaining system security. Ensuring compliance with Oralce license agreement Controlling and monitoring user access to the database. Monitoring and optimizing the performance of the database. Planning for backup and recovery of database information. Maintain archived data on tape Backing up and restoring the database. Contacting Oracle Corporation for technical support.

87. What is a trace file and how is it created ?

Each server and background process can write an associated trace file. When an internal error is detected by a process or user process, it dumps information about the error to its trace. This can be used for tuning the database.

88. What is a profile ?

Each database user is assigned a Profile that specifies limitations on various system resources available to the user.

89. How will you enforce security using stored procedures?

Don't grant user access directly to tables within the application. Instead grant the ability to access the procedures that access the tables. When procedure executed it will execute the privilege of procedures owner. Users cannot access tables except via the procedure.

90. What are the dictionary tables used to monitor a database spaces ?

DBA_FREE_SPACE
DBA_SEGMENTS
DBA_DATA_FILES.

91. What are the roles and user accounts created automatically with the database?

DBA - role Contains all database system privileges.
SYS user account - The DBA role will be assigned to this account. All of the base tables and views for the database's dictionary are store in this schema and are manipulated only by ORACLE. SYSTEM user account - It has all the system privileges for the database and additional tables and views that display administrative information and internal tables and views used by oracle tools are created using this username.

92. What are the minimum parameters should exist in the parameter file (init.ora) ?

DB NAME - Must set to a text string of no more than 8 characters and it will be stored inside the datafiles, redo log files and control files and control file while database creation.
DB_DOMAIN - It is string that specifies the network domain where the database is created. The global database name is identified by setting these parameters
(DB_NAME & DB_DOMAIN) CONTORL FILES - List of control filenames of the database. If name is not mentioned then default name will be used.
DB_BLOCK_BUFFERS - To determine the no of buffers in the buffer cache in SGA.
PROCESSES - To determine number of operating system processes that can be connected to ORACLE concurrently. The value should be 5 (background process) and additional 1 for each user.
ROLLBACK_SEGMENTS - List of rollback segments an ORACLE instance acquires at database startup. Also optionally LICENSE_MAX_SESSIONS,LICENSE_SESSION_WARNING and LICENSE_MAX_USERS.

93. How can we specify the Archived log file name format and destination?

By setting the following values in init.ora file. LOG_ARCHIVE_FORMAT = arch %S/s/T/tarc (%S - Log sequence number and is zero left paded, %s - Log sequence number not padded. %T - Thread number lef-zero-paded and %t - Thread number not padded). The file name created is arch 0001 are if %S is used. LOG_ARCHIVE_DEST = path.

94. What is user Account in Oracle database?

An user account is not a physical structure in Database but it is having important relationship to the objects in the database and will be having certain privileges. 95. When will the data in the snapshot log be used?

We must be able to create a after row trigger on table (i.e., it should be not be already available) After giving table privileges. We cannot specify snapshot log name because oracle uses the name of the master table in the name of the database objects that support its snapshot log. The master table name should be less than or equal to 23 characters. (The table name created will be MLOGS_tablename, and trigger name will be TLOGS name).

96. What dynamic data replication?

Updating or Inserting records in remote database through database triggers. It may fail if remote database is having any problem.

97. What is Two-Phase Commit ?

Two-phase commit is mechanism that guarantees a distributed transaction either commits on all involved nodes or rolls back on all involved nodes to maintain data consistency across the global distributed database. It has two phase, a Prepare Phase and a Commit Phase.

98. How can you Enforce Referential Integrity in snapshots ?

Time the references to occur when master tables are not in use. Peform the reference the manually immdiately locking the master tables. We can join tables in snopshots by creating a complex snapshots that will based on the master tables.

99. What is a SQL * NET?

SQL *NET is ORACLE's mechanism for interfacing with the communication protocols used by the networks that facilitate distributed processing and distributed databases. It is used in Clint-Server and Server-Server communications.

100. What is a SNAPSHOT ?

Snapshots are read-only copies of a master table located on a remote node which is periodically refreshed to reflect changes made to the master table.

101. What is the mechanism provided by ORACLE for table replication ?

Snapshots and SNAPSHOT LOGs

102. What is snapshots?

Snapshot is an object used to dynamically replicate data between distribute database at specified time intervals. In ver 7.0 they are read only.

103. What are the various type of snapshots?

Simple and Complex.

104. Describe two phases of Two-phase commit ?

Prepare phase - The global coordinator (initiating node) ask a participants to prepare (to promise to commit or rollback the transaction, even if there is a failure) Commit - Phase - If all participants respond to the coordinator that they are prepared, the coordinator asks all nodes to commit the transaction, if all participants cannot prepare, the coordinator asks all nodes to roll back the transaction.

105. What is snapshot log ?

It is a table that maintains a record of modifications to the master table in a snapshot. It is stored in the same database as master table and is only available for simple snapshots. It should be created before creating snapshots.

106. What are the benefits of distributed options in databases?

Database on other servers can be updated and those transactions can be grouped together with others in a logical unit.
Database uses a two phase commit.

107. What are the options available to refresh snapshots ?

COMPLETE - Tables are completely regenerated using the snapshots query and the master tables every time the snapshot referenced.
FAST - If simple snapshot used then a snapshot log can be used to send the changes to the snapshot tables.
FORCE - Default value. If possible it performs a FAST refresh; Otherwise it will perform a complete refresh.

108. What is a SNAPSHOT LOG ?

A snapshot log is a table in the master database that is associated with the master table. ORACLE uses a snapshot log to track the rows that have been updated in the master table. Snapshot logs are used in updating the snapshots based on the master table.

109. What is Distributed database ?

A distributed database is a network of databases managed by multiple database servers that appears to a user as single logical database. The data of all databases in the distributed database can be simultaneously accessed and modified.

110. How can we reduce the network traffic?

- Replication of data in distributed environment.
- Using snapshots to replicate data.
- Using remote procedure calls.

111. Differentiate simple and complex, snapshots ?

- A simple snapshot is based on a query that does not contains GROUP BY clauses, CONNECT BY clauses, JOINs, sub-query or snashot of operations.
- A complex snapshots contain atleast any one of the above.

112. What are the Built-ins used for sending Parameters to forms?

You can pass parameter values to a form when an application executes the call_form, New_form, Open_form or Run_product.

113. Can you have more than one content canvas view attached with a window?

Yes. Each window you create must have atleast one content canvas view assigned to it. You can also create a window that has manipulated content canvas view. At run time only one of the content canvas views assign to a window is displayed at a time.

114. Is the After report trigger fired if the report execution fails?

Yes.

115. Does a Before form trigger fire when the parameter form is suppressed?

Yes.

116. Is it possible to split the print reviewer into more than one region?

Yes

117. Is it possible to center an object horizontally in a repeating frame that has a variable horizontal size?

Yes

118. For a field in a repeating frame, can the source come from the column which does not exist in the data group which forms the base for the frame?

Yes

119. Can a field be used in a report without it appearing in any data group?

Yes

120. The join defined by the default data link is an outer join yes or no?

Yes

121. Can a formula column referred to columns in higher group?

Yes

122. Can a formula column be obtained through a select statement?

Yes

123. Is it possible to insert comments into sql statements return in the data model editor?

Yes

124. Is it possible to disable the parameter from while running the report?

Yes

126. When a form is invoked with call_form, Does oracle forms issues a save point?

Yes

127. Can a property clause itself be based on a property clause?

Yes

128. If a parameter is used in a query without being previously defined, what diff. exist betw. report 2.0 and 2.5 when the query is applied?

While both reports 2.0 and 2.5 create the parameter, report 2.5 gives a message that a bind parameter has been created.

129. What are the sql clauses supported in the link property sheet?

Where start with having.

130. What is trigger associated with the timer?

When-timer-expired.

131. What are the trigger associated with image items?

When-image-activated fires when the operators double clicks on an image itemwhen-image-pressed fires when an operator clicks or double clicks on an image item

132. What are the different windows events activated at runtimes?

When_window_activated
When_window_closed
When_window_deactivated
When_window_resized
Within this triggers, you can examine the built in system variable system. event_window to determine the name of the window for which the trigger fired.

133. When do you use data parameter type?

When the value of a data parameter being passed to a called product is always the name of the record group defined in the current form. Data parameters are used to pass data to produts invoked with the run_product built-in subprogram.

134. What is difference between open_form and call_form?

when one form invokes another form by executing open_form the first form remains displayed, and operators can navigate between the forms as desired. when one form invokes another form by executing call_form, the called form is modal with respect to the calling form. That is, any windows that belong to the calling form are disabled, and operators cannot navigate to them until they first exit the called form.

135. What is new_form built-in?

When one form invokes another form by executing new_form oracle form exits the first form and releases its memory before loading the new form calling new form completely replace the first with the second. If there are changes pending in the first form, the operator will be prompted to save them before the new form is loaded.

136. What is the "LOV of Validation" Property of an item? What is the use of it?

When LOV for Validation is set to True, Oracle Forms compares the current value of the text item to the values in the first column displayed in the LOV. Whenever the validation event occurs. If the value in the text item matches one of the values in the first column of the LOV, validation succeeds, the LOV is not displayed, and processing continues normally. If the value in the text item does not match one of the values in the first column of the LOV, Oracle Forms displays the LOV and uses the text item value as the search criteria to automatically reduce the list.

137. What is the diff. when Flex mode is mode on and when it is off?

When flex mode is on, reports automatically resizes the parent when the child is resized.

138. What is the diff. when confine mode is on and when it is off?

When confine mode is on, an object cannot be moved outside its parent in the layout.

139. What are visual attributes?

Visual attributes are the font, color, pattern proprieties that you set for form and menu objects that appear in your application interface.

140. Which of the two views should objects according to possession?

view by structure.

141. What are the two types of views available in the object navigator(specific to report 2.5)?

View by structure and view by type .

142. What are the vbx controls?

Vbx control provide a simple method of building and enhancing user interfaces. The controls can use to obtain user inputs and display program outputs.vbx control where originally develop as extensions for the ms visual basic environments and include such items as sliders, rides and knobs.

143. What is the use of transactional triggers?

Using transactional triggers we can control or modify the default functionality of the oracle forms.

144. How do you create a new session while open a new form?

Using open_form built-in setting the session option Ex. Open_form('Stocks ',active,session). when invoke the mulitiple forms with open form and call_form in the same application, state whether the following are true/False

145. What are the ways to monitor the performance of the report?

Use reports profile executable statement. Use SQL trace facility.

146. If two groups are not linked in the data model editor, What is the hierarchy between them?

Two group that is above are the left most rank higher than the group that is to right or below it.

147. An open form can not be execute the call_form procedure if you chain of called forms has been initiated by another open form?

True

148. Explain about horizontal, Vertical tool bar canvas views?

Tool bar canvas views are used to create tool bars for individual windows. Horizontal tool bars are display at the top of a window, just under its menu bar. Vertical Tool bars are displayed along the left side of a window

149. What is the purpose of the product order option in the column property sheet?

To specify the order of individual group evaluation in a cross products.

150. What is the use of image_zoom built-in?

To manipulate images in image items.

151. How do you reference a parameter indirectly?

To indirectly reference a parameter use the NAME IN, COPY 'built-ins to indirectly set and reference the parameters value' Example name_in ('capital parameter my param'), Copy ('SURESH','Parameter my_param')

152. What is a timer?

Timer is an "internal time clock" that you can programmatically create to perform an action each time the times.

153. What are the two phases of block coordination?

There are two phases of block coordination: the clear phase and the population phase. During, the clear phase, Oracle Forms navigates internally to the detail block and flushes the obsolete detail records. During the population phase, Oracle Forms issues a SELECT statement to repopulate the detail block with detail records associated with the new master record. These operations are accomplished through the execution of triggers.

154. What are Most Common types of Complex master-detail relationships?

There are three most common types of complex master-detail relationships:
master with dependent details
master with independent details
detail with two masters

155. What is a text list?

The text list style list item appears as a rectangular box which displays the fixed number of values. When the text list contains values that can not be displayed, a vertical scroll bar appears, allowing the operator to view and select undisplayed values.

156. What is term?

The term is terminal definition file that describes the terminal form which you are using r20run.

157. What is use of term?

The term file which key is correspond to which oracle report functions.

158. What is pop list?

The pop list style list item appears initially as a single field (similar to a text item field). When the operator selects the list icon, a list of available choices appears.

159. What is the maximum no of chars the parameter can store?

The maximum no of chars the parameter can store is only valid for char parameters, which can be upto 64K. No parameters default to 23Bytes and Date parameter default to 7Bytes.

160. What are the default extensions of the files created by library module?

The default file extensions indicate the library module type and storage format .pll - pl/sql library module binary

161. What are the Coordination Properties in a Master-Detail relationship?

The coordination properties are
Deferred
Auto-Query
These Properties determine when the population phase of block
coordination should occur.

162. How do you display console on a window ?

The console includes the status line and message line, and is displayed at the bottom of the window to which it is assigned.To specify that the console should be displayed, set the console window form property to the name of any window in the form. To include the console, set console window to Null.

163. What are the different Parameter types?

Text ParametersData Parameters

164. State any three mouse events system variables?

System.mouse_button_pressedSystem.mouse_button_shift

165. What are the types of calculated columns available?

Summary, Formula, Placeholder column.

166. Explain about stacked canvas views?

Stacked canvas view is displayed in a window on top of, or "stacked" on the content canvas view assigned to that same window. Stacked canvas views obscure some part of the underlying content canvas view, and or often shown and hidden programmatically.

167. What are the built_ins used the display the LOV?

Show_lov
List_values

168. What is the difference between SHOW_EDITOR and EDIT_TEXTITEM?

Show editor is the generic built-in which accepts any editor name and takes some input string and returns modified output string. Whereas the edit_textitem built-in needs the input focus to be in the text item before the built-in is executed.

169. What are the built-ins that are used to Attach an LOV programmatically to an item?

set_item_property
get_item_property
(by setting the LOV_NAME property)

170. How do you call other Oracle Products from Oracle Forms?

Run_product is a built-in, Used to invoke one of the supported oracle tools products and specifies the name of the document or module to be run. If the called product is unavailable at the time of the call, Oracle Forms returns a message to the operator.

171. What is the main diff. bet. Reports 2.0 & Reports 2.5?

Report 2.5 is object oriented.

172. What are the different file extensions that are created by oracle reports?

Rep file and Rdf file.

173. What is strip sources generate options?

Removes the source code from the library file and generates a library files that contains only pcode. The resulting file can be used for final deployment, but can not be subsequently edited in the designer.ex. f45gen module=old_lib.pll userid=scott/tiger strip_source YES output_file

176. What is the basic data structure that is required for creating an LOV?

Record Group.

177. What is the Maximum allowed length of Record group Column?

Record group column names cannot exceed 30 characters.

178. Which parameter can be used to set read level consistency across multiple queries?

Read only

179. What are the different types of Record Groups?

Query Record Groups
NonQuery Record Groups
State Record Groups

180. From which designation is it preferred to send the output to the printed?

Previewer

181. what are difference between post database commit and post-form commit?

Post-form commit fires once during the post and commit transactions process, after the database commit occurs. The post-form-commit trigger fires after inserts, updates and deletes have been posted to the database but before the transactions have been finalized in the issuing the command. The post-database-commit trigger fires after oracle forms issues the commit to finalized transactions.

182. What are the different display styles of list items?

Pop_listText_listCombo box

183. Which of the above methods is the faster method?

performing the calculation in the query is faster.

184. With which function of summary item is the compute at options required?

percentage of total functions.

185. What are parameters?

Parameters provide a simple mechanism for defining and setting the valuesof inputs that are required by a form at startup. Form parameters are variables of type char,number,date that you define at design time.

186. What are the three types of user exits available ?

Oracle Precompiler exits, Oracle call interface, NonOracle user exits.

187. How many windows in a form can have console?

Only one window in a form can display the console, and you cannot change the console assignment at runtime.

188.If the maximum record retrieved property of the query is set to 10 then a summary value will be calculated?

Only for 10 records.

189.What are the two repeating frame always associated with matrix object?

One down repeating frame below one across repeating frame.

190. What are the master-detail triggers?\

On-Check_delete_masterOn_clear_detailsOn_populate_details

191. What are the different objects that you cannot copy or reference in object groups?

Objects of different modules
Another object groups
Individual block dependent items
Program units.

192. What is an OLE?

Object Linking & Embedding provides you with the capability to integrate objects from many Ms-Windows applications into a single compound document creating integrated applications enables you to use the features form .

193. Is it possible to modify an external query in a report which contains it?

No.

194. Does a grouping done for objects in the layout editor affect the grouping done in the data model editor?

No.

195. Can a repeating frame be created without a data group as a base?

No

196. If a break order is set on a column would it affect columns which are under the column?

No

197. Is it possible to set a filter condition in a cross product group in matrix reports?

No

198. Do user parameters appear in the data modal editor in 2.5?

No

199. Can you pass data parameters to forms?

No

200. Is it possible to link two groups inside a cross products after the cross products group has been created?

no

201. What are the different modals of windows?

Modalless windows
Modal windows

202. What are modal windows?

Modal windows are usually used as dialogs, and have restricted functionality compared to modelless windows. On some platforms for example operators cannot resize, scroll or iconify a modal window.

203. What are the different default triggers created when Master Deletes Property is set to Non-isolated?

Master Deletes Property Resulting Triggers
----------------------------------------------------
Non-Isolated(the default) On-Check-Delete-Master
On-Clear-Details
On-Populate-Details

204. What are the different default triggers created when Master Deletes Property is set to isolated?

Master Deletes Property Resulting Triggers
---------------------------------------------------
Isolated On-Clear-Details
On-Populate-Details

205. What are the different default triggers created when Master Deletes Property is set to Cascade?

Master Deletes Property Resulting Triggers
---------------------------------------------------
Cascading On-Clear-Details
On-Populate-Details
Pre-delete

206. What is the diff. bet. setting up of parameters in reports 2.0 reports2.5?

LOVs can be attached to parameters in the reports 2.5 parameter form.

207. What are the difference between lov & list item?

Lov is a property where as list item is an item. A list item can have only one column, lov can have one or more columns.

208. What is the advantage of the library?

Libraries provide a convenient means of storing client-side program units and sharing them among multiple applications. Once you create a library, you can attach it to any other form, menu, or library modules. When you can call library program units from triggers menu items commands and user named routine, you write in the modules to which you have attach the library. When a library attaches another library, program units in the first library can reference program units in the attached library. Library support dynamic loading-that is library program units are loaded into an application only when needed. This can significantly reduce the run-time memory requirements of applications.

209. What is lexical reference? How can it be created?

Lexical reference is place_holder for text that can be embedded in a sql statements. A lexical reference can be created using & before the column or parameter name.

210. What is system.coordination_operation?

It represents the coordination causing event that occur on the master block in master-detail relation.

211. What is synchronize?

It is a terminal screen with the internal state of the form. It updates the screen display to reflect the information that oracle forms has in its internal representation of the screen.

212. What use of command line parameter cmd file?

It is a command line argument that allows you to specify a file that contain a set of arguments for r20run.

213. What is a Text_io Package?

It allows you to read and write information to a file in the file system.

214. What is forms_DDL?

Issues dynamic Sql statements at run time, including server side pl/SQl and DDL

215. How is link tool operation different bet. reports 2 & 2.5?

In Reports 2.0 the link tool has to be selected and then two fields to be linked are selected and the link is automatically created. In 2.5 the first field is selected and the link tool is then used to link the first field to the second field.

216. What are the different styles of activation of ole Objects?

In place activationExternal activation

217. How do you reference a Parameter?

In Pl/Sql, You can reference and set the values of form parameters using bind variables syntax. Ex. PARAMETER name = '' or :block.item = PARAMETER Parameter name

218. What is the difference between object embedding & linking in Oracle forms?

In Oracle forms, Embedded objects become part of the form module, and linked objects are references from a form module to a linked source file.

219. Name of the functions used to get/set canvas properties?

Get_view_property, Set_view_property

220. What are the built-ins that are used for setting the LOV properties at runtime?

get_lov_property
set_lov_property

221. What are the built-ins used for processing rows?

Get_group_row_count(function)
Get_group_selection_count(function)
Get_group_selection(function)
Reset_group_selection(procedure)
Set_group_selection(procedure)
Unset_group_selection(procedure)

222. What are built-ins used for Processing rows?

GET_GROUP_ROW_COUNT(function)
GET_GROUP_SELECTION_COUNT(function)
GET_GROUP_SELECTION(function)
RESET_GROUP_SELECTION(procedure)
SET_GROUP_SELECTION(procedure)
UNSET_GROUP_SELECTION(procedure)

223. What are the built-in used for getting cell values?

Get_group_char_cell(function)
Get_groupcell(function)
Get_group_number_cell(function)

224. What are the built-ins used for Getting cell values?

GET_GROUP_CHAR_CELL (function)
GET_GROUPCELL(function)
GET_GROUP_NUMBET_CELL(function)

225. Atleast how many set of data must a data model have before a data model can be base on it?

Four

226. To execute row from being displayed that still use column in the row which property can be used?

Format trigger.

227. What are different types of modules available in oracle form?

Form module - a collection of objects and code routines Menu modules - a collection of menus and menu item commands that together make up an application menu library module - a collection of user named procedures, functions and packages that can be called from other modules in the application

228. What is the remove on exit property?

For a modelless window, it determines whether oracle forms hides the window automatically when the operators navigates to an item in the another window.

229. What is WHEN-Database-record trigger?

Fires when oracle forms first marks a record as an insert or an update. The trigger fires as soon as oracle forms determines through validation that the record should be processed by the next post or commit as an insert or update. c generally occurs only when the operators modifies the first item in the record, and after the operator attempts to navigate out of the item.

230. What is a difference between pre-select and pre-query?

Fires during the execute query and count query processing after oracle forms constructs the select statement to be issued, but before the statement is actually issued. The pre-query trigger fires just before oracle forms issues the select statement to the database after the operator as define the example records by entering the query criteria in enter query mode.Pre-query trigger fires before pre-select trigger.

231. What are built-ins associated with timers?

find_timercreate_timerdelete_timer

232. What are the built-ins used for finding object ID functions?

Find_group(function)
Find_column(function)

233. What are the built-ins used for finding Object ID function?

FIND_GROUP(function)
FIND_COLUMN(function)

234. Any attempt to navigate programmatically to disabled form in a call_form stack is allowed?

False

235. Use the Add_group_row procedure to add a row to a static record group 1. true or false?

False

236. Use the add_group_column function to add a column to record group that was created at a design time?

False

237. What are the various sub events a mouse double click event involves? What are the various sub events a mouse double click event involves?

Double clicking the mouse consists of the mouse down, mouse up, mouse click, mouse down & mouse up events.

238. How can a break order be created on a column in an existing group? What are the various sub events a mouse double click event involves?

By dragging the column outside the group.

239. What is the use of place holder column? What are the various sub events a mouse double click event involves?

A placeholder column is used to hold calculated values at a specified place rather than allowing is to appear in the actual row where it has to appear.

240. What is the use of hidden column? What are the various sub events a mouse double click event involves?

A hidden column is used to when a column has to embed into boilerplate text.

241. What is the use of break group? What are the various sub events a mouse double click event involves?

A break group is used to display one record for one group ones. While multiple related records in other group can be displayed.

242. What is an anchoring object & what is its use? What are the various sub events a mouse double click event involves?

An anchoring object is a print condition object which used to explicitly or implicitly anchor other objects to itself.

243. What are the various sub events a mouse double click event involves? What are the various sub events a mouse double click event involves?

Double clicking the mouse consists of the mouse down, mouse up, mouse click, mouse down & mouse up events.

245. What are the default parameter that appear at run time in the parameter screen? What are the various sub events a mouse double click event involves?

Destype and Desname.

246. What are the built-ins used for Creating and deleting groups?

CREATE-GROUP (function)
CREATE_GROUP_FROM_QUERY(function)
DELETE_GROUP(procedure)

247. What are different types of canvas views?

Content canvas views
Stacked canvas views
Horizontal toolbar
vertical toolbar.

248. What are the different types of Delete details we can establish in Master-Details?

Cascade
Isolate
Non-isolate

249. What is relation between the window and canvas views?

Canvas views are the back ground objects on which you place the interface items (Text items), check boxes, radio groups etc.,) and boilerplate objects (boxes, lines, images etc.,) that operators interact with us they run your form . Each canvas views displayed in a window.

250. What is a User_exit?

Calls the user exit named in the user_exit_string. Invokes a 3Gl program by name which has been properly linked into your current oracle forms executable.

251. How is it possible to select generate a select set for the query in the query property sheet?

By using the tables/columns button and then specifying the table and the column names.

252. How can values be passed bet. precompiler exits & Oracle call interface?

By using the statement EXECIAFGET & EXECIAFPUT.

253. How can a square be drawn in the layout editor of the report writer?

By using the rectangle tool while pressing the (Constraint) key.

254. How can a text file be attached to a report while creating in the report writer?

By using the link file property in the layout boiler plate property sheet.

255. How can I message to passed to the user from reports?

By using SRW.MESSAGE function.

256. How is possible to restrict the user to a list of values while entering values for parameters?

By setting the Restrict To List property to true in the parameter property sheet.

257. How can a button be used in a report to give a drill down facility?

By setting the action associated with button to Execute pl/sql option and using the SRW.Run_report function.

258. How can a cross product be created?

By selecting the cross products tool and drawing a new group surrounding the base group of the cross products.

259. What are different types of images?

Boiler plate imagesImage Items

260. What is the difference between boiler plat images and image items?

Boiler plate Images are static images (Either vector or bit map) that you import from the file system or database to use a graphical elements in your form, such as company logos and maps. Image items are special types of interface controls that store and display either vector or bitmap images. Like other items that store values, image items can be either base table items(items that relate directly to database columns) or control items. The definition of an image item is stored as part of the form module FMB and FMX files, but no image file is actually associated with an image item until the item is populate at run time.

261. What is bind reference and how can it be created?

Bind reference are used to replace the single value in sql, pl/sql statements a bind reference can be created using a (:) before a column or a parameter name.

262. What are the triggers available in the reports?

Before report, Before form, After form , Between page, After report.

263. Give the sequence of execution of the various report triggers?

Before form , After form , Before report, Between page, After report.

264. Why is a Where clause faster than a group filter or a format trigger?

Because, in a where clause the condition is applied during data retrievalthan after retrieving the data.

265. Why is it preferable to create a fewer no. of queries in the data model?

Because for each query, report has to open a separate cursor and has to rebind, execute and fetch data.

266. Where is the external query executed at the client or the server?

At the server.

267. Where is a procedure return in an external pl/sql library executed at the client or at the server?

At the client.

268. What is coordination Event?

Any event that makes a different record in the master block the current record is a coordination causing event.

269. What is the difference between OLE Server & Ole Container?

An Ole server application creates ole Objects that are embedded or linked in ole Containers ex. Ole servers are ms_word & ms_excel. OLE containers provide a place to store, display and manipulate objects that are created by ole server applications. Ex. oracle forms is an example of an ole Container.

270. What is an object group?

An object group is a container for a group of objects; you define an object group when you want to package related objects, so that you copy or reference them in other modules.

271. What is an LOV?

An LOV is a scrollable popup window that provides the operator with either a single or multi column selection list.

272. At what point of report execution is the before Report trigger fired?

After the query is executed but before the report is executed and the records are displayed.

273. What are the built -ins used for Modifying a groups structure?

ADD-GROUP_COLUMN (function)
ADD_GROUP_ROW (procedure)
DELETE_GROUP_ROW(procedure)

274. What is an user exit used for?

A way in which to pass control (and possibly arguments ) form Oracle report to another Oracle products of 3 GL and then return control ( and ) back to Oracle reports.

275. What is the User-Named Editor?

A user named editor has the same text editing functionality as the default editor, but, because it is a named object, you can specify editor attributes such as windows display size, position, and title.

276. What are the Built-ins to display the user-named editor?

A user named editor can be displayed programmatically with the built in procedure SHOW-EDITOR, EDIT_TETITEM independent of any particular text item.

277. What is a Static Record Group?

A static record group is not associated with a query, rather, you define its structure and row values at design time, and they remain fixed at runtime.

278. What is a record group?

A record group is an internal Oracle Forms that structure that has a column/row framework similar to a database table. However, unlike database tables, record groups are separate objects that belong to the form module which they are defined.

279. How many number of columns a record group can have?

A record group can have an unlimited number of columns of type CHAR, LONG, NUMBER, or DATE provided that the total number of column does not exceed 64K.

280. What is a Query Record Group?

A query record group is a record group that has an associated SELECT statement. The columns in a query record group derive their default names, data types, had lengths from the database columns referenced in the SELECT statement. The records in query record group are the rows retrieved by the query associated with that record group. What is a Non Query Record Group?

281. What is a property clause?

A property clause is a named object that contains a list of properties and their settings. Once you create a property clause you can base other object on it. An object based on a property can inherit the setting of any property in the clause that makes sense for that object.

282. What is a physical page ? & What is a logical page ?

A physical page is a size of a page. That is output by the printer. The logical page is the size of one page of the actual report as seen in the Previewer.

283. What does the term panel refer to with regard to pages?

A panel is the no. of physical pages needed to print one logical page.

284. What is a master detail relationship?

A master detail relationship is an association between two base table blocks- a master block and a detail block. The relationship between the blocks reflects a primary key to foreign key relationship between the tables on which the blocks are based.

285.What is a library?

A library is a collection of subprograms including user named procedures, functions and packages.

286. How can a group in a cross products be visually distinguished from a group that does not form a cross product?

A group that forms part of a cross product will have a thicker border.

287. What is the frame & repeating frame?

A frame is a holder for a group of fields. A repeating frame is used to display a set of records when the no. of records that are to displayed is not known before.

288. What is a combo box?

A combo box style list item combines the features found in list and text item. Unlike the pop list or the text list style list items, the combo box style list item will both display fixed values and accept one operator entered value.

289. What are three panes that appear in the run time pl/sql interpreter?

1. Source pane.
2. interpreter pane.
3. Navigator pane.

290. What are the two panes that Appear in the design time pl/sql interpreter?

1. Source pane.
2. Interpreter pane

291. What are the two ways by which data can be generated for a parameters list of values?

1. Using static values.
2. Writing select statement.

292. What are the various methods of performing a calculation in a report ?

1. Perform the calculation in the SQL statements itself.
2. Use a calculated / summary column in the data model.

293. What are the default extensions of the files created by menu module?

.mmb,
.mmx

294. What are the default extensions of the files created by forms modules?

.fmb - form module binary
.fmx - form module executable

295. To display the page no. for each page on a report what would be the source & logical page no. or & of physical page no.?

& physical page no.

296. It is possible to use raw devices as data files and what is the advantages over file. system files ?

Yes. The advantages over file system files. I/O will be improved because Oracle is bye-passing the kernnel which writing into disk. Disk Corruption will be very less.

297. What are disadvantages of having raw devices ?

We should depend on export/import utility for backup/recovery (fully reliable) The tar command cannot be used for physical file backup, instead we can use dd command which is less flexible and has limited recoveries.

298. What is the significance of having storage clause ?

We can plan the storage for a table as how much initial extents are required, how much can be extended next, how much % should leave free for managing row updations etc.,

299. What is the use of INCTYPE option in EXP command ?

Type export should be performed COMPLETE,CUMULATIVE,INCREMENTAL. List the sequence of events when a large transaction that exceeds beyond its optimal value when an entry wraps and causes the rollback segment toexpand into anotion Completes. e. will be written.

300. What is the use of FILE option in IMP command ?

The name of the file from which import should be performed.

301. What is a Shared SQL pool?

The data dictionary cache is stored in an area in SGA called the Shared SQL Pool. This will allow sharing of parsed SQL statements among concurrent users.

302. What is hot backup and how it can be taken?

Taking backup of archive log files when database is open. For this the ARCHIVELOG mode should be enabled. The following files need to be backed up. All data files. All Archive log, redo log files. All control files.

303. List the Optional Flexible Architecture (OFA) of Oracle database? or How can we organize the tablespaces in Oracle database to have maximum performance ?

SYSTEM - Data dictionary tables.
DATA - Standard operational tables.
DATA2- Static tables used for standard operations
INDEXES - Indexes for Standard operational tables.
INDEXES1 - Indexes of static tables used for standard operations.
TOOLS - Tools table.
TOOLS1 - Indexes for tools table.
RBS - Standard Operations Rollback Segments,
RBS1,RBS2 - Additional/Special Rollback segments.
TEMP - Temporary purpose tablespace
TEMP_USER - Temporary tablespace for users.
USERS - User tablespace.

304. How to implement the multiple control files for an existing database ?

Shutdown the database Copy one of the existing control file to new location Edit Config ora file by adding new control file. name Restart the database.

305. What is advantage of having disk shadowing/ Mirroring ?

Shadow set of disks save as a backup in the event of disk failure. In most Operating System if any disk failure occurs it automatically switchover to place of failed disk. Improved performance because most OS support volume shadowing can direct file I/O request to use the shadow set of files instead of the main set of files. This reduces I/O load on the main set of disks.

306. How will you force database to use particular rollback segment ?

SET TRANSACTION USE ROLLBACK SEGMENT rbs_name.

307. Why query fails sometimes ?

Rollback segment dynamically extent to handle larger transactions entry loads. A single transaction may wipeout all available free space in the Rollback Segment Tablespace. This prevents other user using Rollback segments.

308. What is the use of RECORD LENGTH option in EXP command ?

Record length in bytes.

309. How will you monitor rollback segment status ?

Querying the DBA_ROLLBACK_SEGS view
IN USE - Rollback Segment is on-line.
AVAILABLE - Rollback Segment available but not on-line.
OFF-LINE - Rollback Segment off-line
INVALID - Rollback Segment Dropped.
NEEDS RECOVERY - Contains data but need recovery or corupted.
PARTLY AVAILABLE - Contains data from an unresolved transaction involving a distributed database.

310. What is meant by Redo Log file mirroring ? How it can be achieved?

Process of having a copy of redo log files is called mirroring. This can be achieved by creating group of log files together, so that LGWR will automatically writes them to all the members of the current on-line redo log group. If any one group fails then database automatically switch over to next group. It degrades performance.

311. Which parameter in Storage clause will reduce no. of rows per block?

PCTFREE parameter
Row size also reduces no of rows per block.

312. What is meant by recursive hints ?

Number of times processes repeatedly query the dictionary table is called recursive hints. It is due to the data dictionary cache is too small. By increasing the SHARED_POOL_SIZE parameter we can optimize the size of Data Dictionary Cache.

313. What is the use of PARFILE option in EXP command ?

Name of the parameter file to be passed for export.

314. What is the use of PARFILE option in EXP command ?

Name of the parameter file to be passed for export.

315. What is a logical backup?

Logical backup involves reading a set of database records and writing them into a file. Export utility is used for taking backup and Import utility is used to recover from backup.

316. What is the use of TABLES option in EXP command ?

List of tables should be exported.ze)

317. What is the OPTIMAL parameter?

It is used to set the optimal length of a rollback segment.

318. What is a Rollback segment entry ?

It is the set of before image data blocks that contain rows that are modified by a transaction. Each Rollback Segment entry must be completed within one rollback segment. A single rollback segment can have multiple rollback segment entries.

319. What is mean by Program Global Area (PGA) ?

It is area in memory that is used by a Single Oracle User Process.

320. What is hit ratio ?

It is a measure of well the data cache buffer is handling requests for data. Hit Ratio = (Logical Reads - Physical Reads - Hits Misses)/ Logical Reads.

321. What are the different kind of export backups?

Full back - Complete database
Incremental - Only affected tables from last incremental date/full backup date.
Cumulative backup - Only affected table from the last cumulative date/full backup date.

322. How free extents are managed in Ver 6.0 and Ver 7.0 ?

Free extents cannot be merged together in Ver 6.0.
Free extents are periodically coalesces with the neighboring free extent in Ver 7.0

323. What is the use of RECORD option in EXP command?

For Incremental exports, the flag indirects whether a record will be stores data dictionary tables recording the export.

324. What is the use of ROWS option in EXP command ?

Flag to indicate whether table rows should be exported. If 'N' only DDL statements for the database objects will be created.

325. What is the use of COMPRESS option in EXP command ?

Flag to indicate whether export should compress fragmented segments into single extents.

326. How will you swap objects into a different table space for an existing database ?

Export the user
Perform import using the command imp system/manager file=export.dmp indexfile=newrite.sql.
This will create all definitions into newfile.sql. Drop necessary objects.
Run the script newfile.sql after altering the tablespaces.
Import from the backup for the necessary objects.

327. How does Space allocation table place within a block ?

Each block contains entries as follows
Fixed block header
Variable block header
Row Header,row date (multiple rows may exists)
PCTEREE (% of free space for row updation in future)

328. What are the factors causing the reparsing of SQL statements in SGA?

Due to insufficient Shared SQL pool size. Monitor the ratio of the reloads takes place while executing SQL statements. If the ratio is greater than 1 then increase the SHARED_POOL_SIZE. LOGICAL & PHYSICAL ARCHITECTURE OF DATABASE.

329. What is dictionary cache ?

Dictionary cache is information about the databse objects stored in a data dictionary table.

330. What is a Control file ?

Database overall physical architecture is maintained in a file called control file. It will be used to maintain internal consistency and guide recovery operations. Multiple copies of control files are advisable.

331. What is Database Buffers ?

Database buffers are cache in the SGA used to hold the data blocks that are read from the data segments in the database such as tables, indexes and clusters DB_BLOCK_BUFFERS parameter in INIT.ORA decides the size.

332. How will you create multiple rollback segments in a database ?

Create a database which implicitly creates a SYSTEM Rollback Segment in a SYSTEM tablespace. Create a Second Rollback Segment name R0 in the SYSTEM tablespace. Make new rollback segment available (After shutdown, modify init.ora file and Start database) Create other tablespaces (RBS) for rollback segments. Deactivate Rollback Segment R0 and activate the newly created rollback segments.

333. What is cold backup? What are the elements of it?

Cold backup is taking backup of all physical files after normal shutdown of database. We need to take.
- All Data files.
- All Control files.
- All on-line redo log files.
- The init.ora file (Optional)

334. What is meant by redo log buffer ?

Changes made to entries are written to the on-line redo log files. So that they can be used in roll forward operations during database recoveries. Before writing them into the redo log files, they will first brought to redo log buffers in SGA and LGWR will write into files frequently. LOG_BUFFER parameter will decide the size.

335. How will you estimate the space required by a non-clustered tables?

Calculate the total header size
Calculate the available dataspace per data block
Calculate the combined column lengths of the average row
Calculate the total average row size.
Calculate the average number rows that can fit in a block
Calculate the number of blocks and bytes required for the table.
After arriving the calculation, add 10 % additional space to calculate the initial extent size for a working table.

336. How will you monitor the space allocation ?

By querying DBA_SEGMENT table/view.

337. What is meant by free extent ?

A free extent is a collection of continuous free blocks in tablespace. When a segment is dropped its extents are reallocated and are marked as free.

338. What is the use of IGNORE option in IMP command ?

A flag to indicate whether the import should ignore errors encounter when issuing CREATE commands.

339. What is the use of ANALYSE ( Ver 7) option in EXP command ?

A flag to indicate whether statistical information about the exported objects should be written to export dump file.

340. What is the use of ROWS option in IMP command ?

A flag to indicate whether rows should be imported. If this is set to 'N' then only DDL for database objects will be executed.

341. What is the use of INDEXES option in EXP command ?

A flag to indicate whether indexes on tables will be exported.

342. What is the use of INDEXES option in IMP command ?

A flag to indicate whether import should import index on tables or not.

343. What is the use of GRANT option in EXP command?

A flag to indicate whether grants on databse objects will be exported or not. Value is 'Y' or 'N'.

344. What is the use of GRANT option in IMP command ?

A flag to indicate whether grants on database objects will be imported.

345. What is the use of FULL option in EXP command ?

A flag to indicate whether full databse export should be performed.

346. What is the use of SHOW option in IMP command ?

A flag to indicate whether file content should be displayed or not.

347. What is the use of CONSTRAINTS option in EXP command ?

A flag to indicate whether constraints on table need to be exported.

348. What is the use of CONSISTENT (Ver 7) option in EXP command ?

A flag to indicate whether a read consistent version of all the exported objects should be maintained.

349. What are the different methods of backing up oracle database ?

- Logical Backups
- Cold Backups
- Hot Backups (Archive log)

350. What is the difference between ON-VALIDATE-FIELD trigger and a POST-CHANGE trigger ?

When you changes the Existing value to null, the On-validate field trigger will fire post change trigger will not fire. At the time of execute-query post-change trigger will fire, on-validate field trigger will not fire.

351. When is PRE-QUERY trigger executed ?

When Execute-query or count-query Package procedures are invoked.

352. How do you trap the error in forms 3.0 ?

using On-Message or On-Error triggers.

353. How many pages you can in a single form ?

Unlimited

354. While specifying master/detail relationship between two blocks specifying the join condition is a must ? True or False. ?

True

355. EXIT_FORM is a restricted package procedure ?a. True b. False

True

356. What is the usage of an ON-INSERT,ON-DELETE and ON-UPDATE TRIGGERS ?

These triggers are executes when inserting, deleting and updating operations are performed and can be used to change the default function of insert, delete or update respectively. For Eg, instead of inserting a row in a table an existing row can be updated in the same table.

357. What are the types of Pop-up window ?

the pop-up field editor
pop-up list of values
pop-up pages.
Alert :

358. What is an SQL *FORMS ?

SQL *forms is 4GL tool for developing and executing; Oracle based interactive application.

359. How do you control the constraints in forms ?

Select the use constraint property is ON Block definition screen.
BLOCK

360. What is the difference between restricted and unrestricted package procedure ?

Restricted package procedure that affects the basic functions of SQL * Forms. It cannot used in all triggers except key triggers. Unrestricted package procedure that does not interfere with the basic functions of SQL * Forms it can be used in any triggers.

361. A query fetched 10 records How many times does a PRE-QUERY Trigger and POST-QUERY Trigger will get executed ?

PRE-QUERY fires once.
POST-QUERY fires 10 times.

362. Give the sequence in which triggers fired during insert operations, when the following 3 triggers are defined at the same block level ?

a. ON-INSERT b. POST-INSERT c. PRE-INSERT

363. State the order in which these triggers are executed ?

POST-FIELD,ON-VALIDATE-FIELD,POST-CHANGE and KEY-NEXTFLD. KEY-NEXTFLD,POST-CHANGE, ON-VALIDATE-FIELD, POST-FIELD. g.

364. What the PAUSE package procedure does ?

Pause suspends processing until the operator presses a function key

365. What do you mean by a page ?

Pages are collection of display information, such as constant text and graphics

366. What are the type of User Exits ?

ORACLE Precompliers user exits
OCI (ORACLE Call Interface)
Non-ORACEL user exits.
Page :

367. What is the difference between an ON-VALIDATE-FIELD trigger and a trigger ?

On-validate-field trigger fires, when the field Validation status New or changed. Post-field-trigger whenever the control leaving form the field, it will fire.

368. Can we use a restricted package procedure in ON-VALIDATE-FIELD Trigger ?

No

369. Is a Key startup trigger fires as result of a operator pressing a key explicitly ?

No

370. Can we use GO-BLOCK package in a pre-field trigger ?

No

371. Can we create two blocks with the same name in form 3.0 ?

No

372. What is Post-Block is a. ???

a. Navigational Trigger.
b. Key trigger
c. Transaction Trigger.

373. What does an on-clear-block Trigger fire?

It fires just before SQL * forms the current block.

374. Name the two files that are created when you generate the form give the filex extension ?

INP (Source File)
FRM (Executable File)

375. What package procedure used for invoke sql *plus from sql *forms ?

Host (E.g. Host (sqlplus))

376. What is the significance of PAGE 0 in forms 3.0 ?

Hide the fields for internal calculation.

377. What are the different types of key triggers ?

Function Key
Key-function
Key-others
Key-startup

378. What is the difference between a Function Key Trigger and Key Function Trigger ?

Function key triggers are associated with individual SQL*FORMS function keys You can attach Key function triggers to 10 keys or key sequences that normally do not perform any SQL * FORMS operations. These keys referred as key F0 through key F9.

379. Committed block sometimes refer to a BASE TABLE ?

False

380. Error_Code is a package proecdure ?

a. True b. false
False

381. How can you execute the user defined triggers in forms 3.0 ?

Execute Trigger (trigger-name)

382. What ERASE package procedure does ?

Erase removes an indicated global variable.

383. What Enter package procedure does ?

Enter Validate-data in the current validation unit.

384. What is the difference between NAME_IN and COPY ?

Copy is package procedure and writes values into a field.
Name in is a package function and returns the contents of the variable to which you apply.

385. What package procedure is used for calling another form ?

Call (E.g. Call(formname)

386. When the form is running in DEBUG mode, If you want to examine the values of global variables and other form variables, What package procedure command you would use in your trigger text ?

Break.
SYSTEM VARIABLES

387. The value recorded in system.last_record variable is of type
a. Number
b. Boolean
c. Character. ?

b. Boolean.

388. What are the unrestricted procedures used to change the popup screen position during run time ?

Anchor-view
Resize -View
Move-View.

389. What is an Alert ?

An alert is window that appears in the middle of the screen overlaying a portion of the current display.

390. Deleting a page removes information about all the fields in that page ? a. True. b. False?

a. True.

391. Two popup pages can appear on the screen at a time ?Two popup pages can appear on the screen at a time ? a. True. b. False?

a. True.

392. Classify the restricted and unrestricted procedure from the following.
a. Call
b. User-Exit
c. Call-Query
d. Up
e. Execute-Query
f. Message
g. Exit-From
h. Post
i. Break?

a. Call - unrestricted
b. User Exit - Unrestricted
c. Call_query - Unrestricted
d. Up - Restricted
e. Execute Query - Restricted
f. Message - Restricted
g. Exit_form - Restricted
h. Post - Restricted
i. Break - Unrestricted.

393. What is an User Exits ?

A user exit is a subroutine which are written in programming languages using pro*C pro *Cobol , etc., that link into the SQL * forms executable.

394. What is a Trigger ?

A piece of logic that is executed at or triggered by a SQL *forms event.

395. What is a Package Procedure ?

A Package procedure is built in PL/SQL procedure.

398. What is the maximum size of a form ?

255 character width and 255 characters Length.

399. What is the difference between system.current_field and system.cursor_field ?

1. System.current_field gives name of the field.
2. System.cursor_field gives name of the field with block name.

400. List the system variables related in Block and Field?

1. System.block_status
2. System.current_block
3. System.current_field
4. System.current_value
5. System.cursor_block
6. System.cursor_field
7. System.field_status.

401. What are the different types of Package Procedure ?

1. Restricted package procedure.
2. Unrestricted package procedure.

402. What are the types of TRIGGERS ?

1. Navigational Triggers.
2. Transaction Triggers.

403. Identify package function from the following ? 1. Error-Code
2. Break
3. Call
4. Error-text
5. Form-failure
6. Form-fatal
7. Execute-query
8. Anchor View
9. Message_code?

1. Error_Code
2. Error_Text
3. Form_Failure
4. Form_Fatal
5. Message_Code

403. Can you attach an lov to a field at run-time? if yes, give the build-in name.?

Yes. Set_item_proprety

404. Is it possible to attach same library to more than one form?

Yes

405. Can you attach an lov to a field at design time?

Yes

406. List the windows event triggers available in Forms 4.0?

When-window-activated,
when-window-closed,
when-window-deactivated,
when-window-resized

407. What are the triggers associated with the image item?

When-Image-activated(Fires when the operator double clicks on an image Items)
When-image-pressed(fires when the operator selects or deselects the image item)

408. What is a visual attribute?

Visual Attributes are the font, color and pattern characteristics of objects that operators see and intract with in our application.

409. How many maximum number of radio buttons can you assign to a radio group?

Unlimited no of radio buttons can be assigned to a radio group

410. How do you pass the parameters from one form to another form?

To pass one or more parameters to a called form, the calling form must perform the following steps in a trigger or user named routine execute the create_parameter_list built-in function to programmatically. Create a parameter list to execute the add parameter built-in procedure to add one or more parameters list. Execute the call_form, New_form or run_product built_in procedure and include the name or id of the parameter list to be passed to the called form.

411. What is a Layout Editor?

The Layout Editor is a graphical design facility for creating and arranging items and boilerplate text and graphics objects in your application's interface.

412. List the Types of Items?

Text item.
Chart item.
Check box.
Display item.
Image item.
List item.
Radio Group.
User Area item.

413. List system variables available in forms 4.0, and not available in forms 3.0?

System.cordination_operation
System Date_threshold
System.effective_Date
System.event_window
System.suppress_working

414. What are the display styles of an alert?

Stop, Caution, note

415. What built-in is used for showing the alert during run-time?

Show_alert.

416. What built-in is used for changing the properties of the window dynamically?

Set_window_property
Canvas-View

417. What are the different types of windows?

Root window, secondary window.

418. What is a predefined exception available in forms 4.0?

Raise form_trigger_failure

419. What is a radio Group?

Radio groups display a fixed no of options that are mutually Exclusive. User can select one out of n number of options.

419. What are the different type of a record group?

Query record group
Static record group
Non query record group

420. What are the menu items that oracle forms 4.0 supports?

Plain, Check,Radio, Separator, Magic

421. Give the equivalent term in forms 4.0 for the following. Page, Page 0?

Page - Canvas-View
Page 0 - Canvas-view null.

422. What triggers are associated with the radio group?

Only when-radio-changed trigger associated with radio group
Visual Attributes.

423. What are the triggers associated with a check box?

Only When-checkbox-activated Trigger associated with a Check box.

424.Can you attach an alert to a field?

No

425. Can a root window be made modal?

No

426. What is a list item?

It is a list of text elements.

427. List some built-in routines used to manipulate images in image_item?

Image_add
Image_and
Image_subtract
Image_xor
Image_zoom

428. Can you change the alert messages at run-time?

If yes, give the name of the built-in to change the alert messages at run-time. Yes. Set_alert_property.

429. What is the built-in used to get and set lov properties during run-time?

Get_lov_property
Set_lov_property
Record Group

430. What is the built-in routine used to count the no of rows in a group?

Get_group _row_count
System Variables

431. Give the Types of modules in a form?

Form
Menu
Library

432. Write the Abbreviation for the following File Extension 1. FMB 2. MMB 3. PLL?

FMB ----- Form Module Binary.
MMB ----- Menu Module Binary.
PLL ------ PL/SQL Library Module Binary.

433. List the built-in routine for controlling window during run-time?

Find_window,
get_window_property,
hide_window,
move_window,
resize_window,
set_window_property,
show_View

434. List the built-in routine for controlling window during run-time?

Find_canvas
Get-Canvas_property
Get_view_property
Hide_View
Replace_content_view
Scroll_view
Set_canvas_property
Set_view_property
Show_view
Alert

435. What is the built-in function used for finding the alert?

Find_alert
Editors

436. List the editors availables in forms 4.0?

Default editor
User_defined editors
system editors.

437. What buil-in routines are used to display editor dynamicaly?

Edit_text item
show_editor
LOV

438. What is an Lov?

A list of values is a single or multi column selection list displayed in a pop-up window

439. What is a record Group?

A record group is an internal oracle forms data structure that has a similar column/row frame work to a database table

440. Give built-in routine related to a record groups?

Create_group (Function)
Create_group_from_query(Function)
Delete_group(Procedure)
Add_group_column(Function)
Add_group_row(Procedure)
Delete_group_row(Procedure)
Populate_group(Function)
Populate_group_with_query(Function)
Set_group_Char_cell(procedure)

441. List the built-in routines for the controlling canvas views during run-time?

Find_canvas
Get-Canvas_property
Get_view_property
Hide_View
Replace_content_view
Scroll_view
Set_canvas_property
Set_view_property
Show_view
Alert

442. System.effective_date system variable is read only True/False?

False

443. What are the built_in used to trapping errors in forms 4?

Error_type return character
Error_code return number
Error_text return char
Dbms_error_code return no.
Dbms_error_text return char

444. what is a display item?

Display items are similar to text items but store only fetched or assigned values. Operators cannot navigate to a display item or edit the value it contains.

445. What are the design facilities available in forms 4.0?

Default Block facility.
Layout Editor.
Menu Editor.
Object Lists.
Property Sheets.
PL/SQL Editor.
Tables Columns Browser.
Built-ins Browser.

446. What are the types of visual attribute settings?

Custom Visual attributes Default visual attributes Named Visual attributes. Window

447. What are the types of canvas-views?

Content View, Stacked View.

448. What are the two ways to incorporate images into a oracle forms application?

Boilerplate Images
Image_items

449. What do you mean by a block in forms4.0?

Block is a single mechanism for grouping related items into a functional unit for storing, displaying and manipulating records.

450. Explain types of Block in forms4.0?

Base table Blocks.
Control Blocks.
1. A base table block is one that is associated with a specific database table or view.
2. A control block is a block that is not associated with a database table. ITEMS

451. What is an Alert?

An alert is a modal window that displays a message notifies the operator of some application condition

455. What are the built-in routines is available in forms 4.0 to create and manipulate a parameter list?

Add_parameter
Create_Parameter_list
Delete_parameter
Destroy_parameter_list
Get_parameter_attr
Get_parameter_list
set_parameter_attr

456 .What is a record Group?

A record group is an internal oracle forms data structure that has a similar column/row frame work to a database table

457 What is a Navigable item?

A navigable item is one that operators can navigate to with the keyboard during default navigation, or that Oracle forms can navigate to by executing a navigational built-in procedure.

458. What is a library in Forms 4.0?

A library is a collection of Pl/SQL program units, including user named procedures, functions & packages

460. How image_items can be populate to field in forms 4.0?

A fetch from a long raw database column PL/Sql assignment to executing the read_image_file built_in procedure to get an image from the file system.

461. What is the content view and stacked view?

A content view is the "Base" view that occupies the entire content pane of the window in which it is displayed. A stacked view differs from a content canvas view in that it is not the base view for the window to which it is assigned

462. What is a Check Box?

A Check Box is a two state control that indicates whether a certain condition or value is on or off, true or false. The display state of a check box is always either "checked" or "unchecked".

463. What is a canvas-view?

A canvas-view is the background object on which you layout the interface items (text-items, check boxes, radio groups, and so on.) and boilerplate objects that operators see and interact with as they run your form. At run-time, operators can see only those items that have been assigned to a specific canvas. Each canvas, in term, must be displayed in a specific window.

464. Explain the following file extension related to library?

.pll,.lib,.pld
The library pll files is a portable design file comparable to an fmb form file
The library lib file is a plat form specific, generated library file comparable to a fmx form file
The pld file is Txt format file and can be used for source controlling your library files Parameter

465. Explain the usage of WHERE CURRENT OF clause in cursors ?

WHERE CURRENT OF clause in an UPDATE,DELETE statement refers to the latest row fetched from a cursor. Database Triggers

466. Name the tables where characteristics of Package, procedure and functions are stored ?

User_objects, User_Source and User_error.

467. Explain the two type of Cursors ?

There are two types of cursors, Implicit Cursor and Explicit Cursor. PL/SQL uses Implicit Cursors for queries. User defined cursors are called Explicit Cursors. They can be declared and used.

468. What are two parts of package ?

The two parts of package are PACKAGE SPECIFICATION & PACKAGE BODY. Package Specification contains declarations that are global to the packages and local to the schema. Package Body contains actual procedures and local declaration of the procedures and cursor declarations.

469. What are two virtual tables available during database trigger execution ?

The table columns are referred as OLD.column_name and NEW.column_name.
For triggers related to INSERT only NEW.column_name values only available.
For triggers related to UPDATE only OLD.column_name NEW.column_name values only available.
For triggers related to DELETE only OLD.column_name values only available.

470. What is Overloading of procedures ?

The Same procedure name is repeated with parameters of different datatypes and parameters in different positions, varying number of parameters is called overloading of procedures. e.g. DBMS_OUTPUT put_line

471. What is a package ? What are the advantages of packages ? What is Pragma EXECPTION_INIT ? Explain the usage ?

The PRAGMA EXECPTION_INIT tells the complier to associate an exception with an oracle error. To get an error message of a specific oracle error. e.g. PRAGMA EXCEPTION_INIT (exception name, oracle error number)

472. What are the return values of functions SQLCODE and SQLERRM ? What is Pragma EXECPTION_INIT ? Explain the usage ?

SQLCODE returns the latest code of the error that has occurred.
SQLERRM returns the relevant error message of the SQLCODE.

473. What are the datatypes a available in PL/SQL ?

Some scalar data types such as NUMBER, VARCHAR2, DATE, CHAR, LONG, BOOLEAN. Some composite data types such as RECORD & TABLE.

474. What is Raise_application_error ?

Raise_application_error is a procedure of package DBMS_STANDARD which allows to issue an user_defined error messages from stored sub-program or database trigger.

475. What are the two parts of a procedure ?

Procedure Specification and Procedure Body.

476. Give the structure of the procedure ?

PROCEDURE name (parameter list.....)
is
local variable declarations
BEGIN
Executable statements.
Exception.
exception handlers
end;

477. What is the basic structure of PL/SQL ?

PL/SQL uses block structure as its basic structure. Anonymous blocks or nested blocks can be used in PL/SQL.

478. Question What is PL/SQL ?

PL/SQL is a procedural language that has both interactive SQL and procedural programming language constructs such as iteration, conditional branching.

479. What is PL/SQL table ?

Objects of type TABLE are called "PL/SQL tables", which are modeled as (but not the same as) database tables, PL/SQL tables use a primary PL/SQL tables can have one column and a primary key. Cursors

480. What happens if a procedure that updates a column of table X is called in a database trigger of the same table ?

Mutation of table occurs.

481. Is it possible to use Transaction control Statements such a ROLLBACK or COMMIT in Database Trigger ? Why ?

It is not possible. As triggers are defined for each table, if you use COMMIT of ROLLBACK in a trigger, it affects logical transaction processing.

482. How many types of database triggers can be specified on a table ? What are they ?

Insert Update Delete
Before Row o.k. o.k. o.k.
After Row o.k. o.k. o.k.
Before Statement o.k. o.k. o.k.
After Statement o.k. o.k. o.k.
If FOR EACH ROW clause is specified, then the trigger for each Row affected by the statement.
If WHEN clause is specified, the trigger fires according to the returned Boolean value.

483. What are the modes of parameters that can be passed to a procedure ?

IN,OUT,IN-OUT parameters.

484. Where the Pre_defined_exceptions are stored ?

In the standard package.
Procedures, Functions & Packages ;

485. Write the order of precedence for validation of a column in a table ?
I. done using Database triggers.
ii. done using Integarity Constraints.?

I & ii.

486. Give the structure of the function ?

FUNCTION name (argument list .....) Return datatype is
local variable declarations
Begin
executable statements
Exception
execution handlers
End;

487. Explain how procedures and functions are called in a PL/SQL block ?

Function is called as part of an expression.
sal := calculate_sal ('a822');
procedure is called as a PL/SQL statement
calculate_bonus ('A822');

488. What are advantages fo Stored Procedures?

Extensibility,Modularity, Reusability, Maintainability and one time compilation.

489. What is an Exception ? What are types of Exception ?

Exception is the error handling part of PL/SQL block. The types are Predefined and user defined. Some of Predefined exceptions are.
CURSOR_ALREADY_OPEN
DUP_VAL_ON_INDEX
NO_DATA_FOUND
TOO_MANY_ROWS
INVALID_CURSOR
INVALID_NUMBER
LOGON_DENIED
NOT_LOGGED_ON
PROGRAM-ERROR
STORAGE_ERROR
TIMEOUT_ON_RESOURCE
VALUE_ERROR
ZERO_DIVIDE
OTHERS.

490. What are the PL/SQL Statements used in cursor processing ?

DECLARE CURSOR cursor name, OPEN cursor name, FETCH cursor name INTO or Record types, CLOSE cursor name.

491. What are the components of a PL/SQL Block ?

Declarative part, Executable part and Exception part.
Datatypes PL/SQL

492. What is a database trigger ? Name some usages of database trigger ?

Database trigger is stored PL/SQL program unit associated with a specific database table. Usages are Audit data modifications, Log events transparently, Enforce complex business rules Derive column values automatically, Implement complex security authorizations. Maintain replicate tables.

493. What is a cursor ? Why Cursor is required ?

Cursor is a named private SQL area from where information can be accessed. Cursors are required to process rows individually for queries returning multiple rows.

494. What is a cursor for loop ?

Cursor for loop implicitly declares %ROWTYPE as loop index,opens a cursor, fetches rows of values from active set into fields in the record and closes when all the records have been processed.
eg. FOR emp_rec IN C1 LOOP
salary_total := salary_total +emp_rec sal;
END LOOP;

495. What will happen after commit statement ?

Cursor C1 is
Select empno,
ename from emp;
Begin
open C1; loop
Fetch C1 into
eno.ename;
Exit When
C1 %notfound;-----
commit;
end loop;
end;
The cursor having query as SELECT .... FOR UPDATE gets closed after COMMIT/ROLLBACK.
The cursor having query as SELECT.... does not get closed even after COMMIT/ROLLBACK.

496. How packaged procedures and functions are called from the following?
a. Stored procedure or anonymous block
b. an application program such a PRC *C, PRO* COBOL
c. SQL *PLUS??

a. PACKAGE NAME.PROCEDURE NAME (parameters);
variable := PACKAGE NAME.FUNCTION NAME (arguments);
EXEC SQL EXECUTE
b.BEGIN
PACKAGE NAME.PROCEDURE NAME (parameters)
variable := PACKAGE NAME.FUNCTION NAME (arguments);
END;
END EXEC;
c. EXECUTE PACKAGE NAME.PROCEDURE if the procedures does not have any out/in-out parameters. A function can not be called.

497. What is a stored procedure ?

A stored procedure is a sequence of statements that perform specific function.

498. What are the components of a PL/SQL block ?

A set of related declarations and procedural statements is called block.

499. What is difference between a PROCEDURE & FUNCTION ?

A FUNCTION is always returns a value using the return statement.
A PROCEDURE may return one or more values through parameters or may not return at all.

500. What is difference between a Cursor declared in a procedure and Cursor declared in a package specification ?

A cursor declared in a package specification is global and can be accessed by other procedures or procedures in a package.
A cursor declared in a procedure is local to the procedure that can not be accessed by other procedures.

501. What are the cursor attributes used in PL/SQL ?

%ISOPEN - to check whether cursor is open or not
% ROWCOUNT - number of rows fetched/updated/deleted.
% FOUND - to check whether cursor has fetched any row. True if rows are fetched.
% NOT FOUND - to check whether cursor has fetched any row. True if no rows are featched.
These attributes are proceeded with SQL for Implicit Cursors and with Cursor name for Explicit Cursors.

502. What are % TYPE and % ROWTYPE ? What are the advantages of using these over datatypes?

% TYPE provides the data type of a variable or a database column to that variable.
% ROWTYPE provides the record type that represents a entire row of a table or view or columns selected in the cursor.
The advantages are :
I. Need not know about variable's data type
ii. If the database definition of a column in a table changes, the data type of a variable changes accordingly.

503. What is difference between % ROWTYPE and TYPE RECORD ?

% ROWTYPE is to be used whenever query returns a entire row of a table or view.
TYPE rec RECORD is to be used whenever query returns columns of different table or views and variables.
E.g. TYPE r_emp is RECORD (eno emp.empno% type,ename emp ename %type );
e_rec emp% ROWTYPE
cursor c1 is select empno,deptno from emp;
e_rec c1 %ROWTYPE.

504. What are the different types of PL/SQL program units that can be defined and stored in ORACLE database ?

Procedures and Functions,Packages and Database Triggers.

505. What are the advantages of having a Package ?

Increased functionality (for example,global package variables can be declared and used by any proecdure in the package) and performance (for example all objects of the package are parsed compiled, and loaded into memory once)

506. What are the uses of Database Trigger ?

Database triggers can be used to automatic data generation, audit data modifications, enforce complex Integrity constraints, and customize complex security authorizations.

507. What is a Procedure ?

A Procedure consist of a set of SQL and PL/SQL statements that are grouped together as a unit to solve a specific problem or perform a set of related tasks.

508. What is a Package ?

A Package is a collection of related procedures, functions, variables and other package constructs together as a unit in the database.

509. What is difference between Procedures and Functions ?

A Function returns a value to the caller where as a Procedure does not.

510. What is Database Trigger ?

A Database Trigger is procedure (set of SQL and PL/SQL statements) that is automatically executed as a result of an insert in,update to, or delete from a table.

511. Can the default values be assigned to actual parameters?

Yes

512. Can a primary key contain more than one columns?

Yes

513. What is an UTL_FILE.What are different procedures and functions associated with it?

UTL_FILE is a package that adds the ability to read and write to operating system files. Procedures associated with it are FCLOSE, FCLOSE_ALL and 5 procedures to output data to a file PUT, PUT_LINE, NEW_LINE, PUTF, FFLUSH.PUT, FFLUSH.PUT_LINE,FFLUSH.NEW_LINE. Functions associated with it are FOPEN, ISOPEN.

514. What are ORACLE PRECOMPILERS?

Using ORACLE PRECOMPILERS, SQL statements and PL/SQL blocks can be contained inside 3GL programs written in C,C++,COBOL,PASCAL, FORTRAN,PL/1 AND ADA. The Precompilers are known as Pro*C,Pro*Cobol,... This form of PL/SQL is known as embedded pl/sql,the language in which pl/sql is embedded is known as the host language. The prcompiler translates the embedded SQL and pl/sql ststements into calls to the precompiler runtime library.The output must be compiled and linked with this library to creater an executable.

515. Differentiate between TRUNCATE and DELETE?

TRUNCATE deletes much faster than DELETE
TRUNCATE
DELETE
It is a DDL statement
It is a DML statement
It is a one way trip,cannot ROLLBACK
One can Rollback
Doesn't have selective features (where clause)
Has
Doesn't fire database triggers
Does
It requires disabling of referential constraints.

516. What is difference between a formal and an actual parameter?

The variables declared in the procedure and which are passed, as arguments are called actual, the parameters in the procedure declaration. Actual parameters contain the values that are passed to a procedure and receive results. Formal parameters are the placeholders for the values of actual parameters

517. What should be the return type for a cursor variable.Can we use a scalar data type as return type?

The return type for a cursor must be a record type.It can be declared explicitly as a user-defined or %ROWTYPE can be used. eg TYPE t_studentsref IS REF CURSOR RETURN students%ROWTYPE

518. What are different Oracle database objects?

-TABLES
-VIEWS
-INDEXES
-SYNONYMS
-SEQUENCES
-TABLESPACES etc

519. What is difference between SUBSTR and INSTR?

SUBSTR returns a specified portion of a string eg SUBSTR('BCDEF',4) output BCDE INSTR provides character position in which a pattern is found in a string. eg INSTR('ABC-DC-F','-',2) output 7 (2nd occurence of '-')

520. Display the number value in Words?

SQL> select sal, (to_char(to_date(sal,'j'), 'jsp'))
from emp;
the output like,
SAL (TO_CHAR(TO_DATE(SAL,'J'),'JSP'))
--------- ----------------------------------------
800 eight hundred
1600 one thousand six hundred
1250 one thousand two hundred fifty
If you want to add some text like, Rs. Three Thousand only.
SQL> select sal "Salary ",
(' Rs. '|| (to_char(to_date(sal,'j'), 'Jsp'))|| ' only.'))
"Sal in Words" from emp
/
Salary Sal in Words
------- -----------------------------------------------
800 Rs. Eight Hundred only.
1600 Rs. One Thousand Six Hundred only.
1250 Rs. One Thousand Two Hundred Fifty only.

521. What is difference between SQL and SQL*PLUS?

SQL*PLUS is a command line tool where as SQL and PL/SQL language interface and reporting tool. Its a command line tool that allows user to type SQL commands to be executed directly against an Oracle database. SQL is a language used to query the relational database(DML,DCL,DDL). SQL*PLUS commands are used to format query result, Set options, Edit SQL commands and PL/SQL.

522. What are various joins used while writing SUBQUERIES?

Self join-Its a join foreign key of a table references the same table. Outer Join--Its a join condition used where One can query all the rows of one of the tables in the join condition even though they don't satisfy the join condition.
Equi-join--Its a join condition that retrieves rows from one or more tables in which one or more columns in one table are equal to one or more columns in the second table.

523. What a SELECT FOR UPDATE cursor represent.?

SELECT......FROM......FOR......UPDATE[OF column-reference][NOWAIT]
The processing done in a fetch loop modifies the rows that have been retrieved by the cursor. A convenient way of modifying the rows is done by a method with two parts: the FOR UPDATE clause in the cursor declaration, WHERE CURRENT OF CLAUSE in an UPDATE or declaration statement.

524. What are various privileges that a user can grant to another user?

-SELECT
-CONNECT
-RESOURCES

525. Display the records between two range?

select rownum, empno, ename from emp where rowid in (select rowid from emp where rownum <=&upto minus select rowid from emp where rownum<&Start);

526. minvalue.sql Select the Nth lowest value from a table?

select level, min('col_name') from my_table where level = '&n' connect by prior ('col_name') < 'col_name')
group by level;
Example:
Given a table called emp with the following columns:
-- id number
-- name varchar2(20)
-- sal number
--
-- For the second lowest salary:
-- select level, min(sal) from emp
-- where level=2
-- connect by prior sal < sal
-- group by level

527. What is difference between Rename and Alias?

Rename is a permanent name given to a table or column whereas Alias is a temporary name given to a table or column which do not exist once the SQL statement is executed.

528. Difference between an implicit & an explicit cursor.?

only one row. However,queries that return more than one row you must declare an explicit cursor or use a cursor FOR loop. Explicit cursor is a cursor in which the cursor name is explicitly assigned to a SELECT statement via the CURSOR...IS statement. An implicit cursor is used for all SQL statements Declare, Open, Fetch, Close. An explicit cursors are used to process multirow SELECT statements An implicit cursor is used to process INSERT, UPDATE, DELETE and single row SELECT. .INTO statements.

529. What is a OUTER JOIN?

Outer Join--Its a join condition used where you can query all the rows of one of the tables in the join condition even though they dont satisfy the join condition.

530. What is a cursor?

Oracle uses work area to execute SQL statements and store processing information PL/SQL construct called a cursor lets you name a work area and access its stored information A cursor is a mechanism used to fetch more than one row in a Pl/SQl block.

531. What is the purpose of a cluster?

Oracle does not allow a user to specifically locate tables, since that is a part of the function of the RDBMS. However, for the purpose of increasing performance, oracle allows a developer to create a CLUSTER. A CLUSTER provides a means for storing data from different tables together for faster retrieval than if the table placement were left to the RDBMS.

532. What is OCI. What are its uses?

Oracle Call Interface is a method of accesing database from a 3GL program. Uses--No precompiler is required,PL/SQL blocks are executed like other DML statements.
The OCI library provides
--functions to parse SQL statemets
--bind input variables
--bind output variables
--execute statements
--fetch the results

533. How you open and close a cursor variable.Why it is required?

OPEN cursor variable FOR SELECT...Statement
CLOSE cursor variable In order to associate a cursor variable with a particular SELECT statement OPEN syntax is used. In order to free the resources used for the query CLOSE statement is used.

534. Display Odd/ Even number of records?

Odd number of records:
select * from emp where (rowid,1) in (select rowid, mod(rownum,2) from emp);
Output:-
1
3
5
Even number of records:
select * from emp where (rowid,0) in (select rowid, mod(rownum,2) from emp)
Output:-
2
4
6

535. What are various constraints used in SQL?

-NULL
-NOT NULL
-CHECK
-DEFAULT

536. Can cursor variables be stored in PL/SQL tables.If yes how. If not why?

No, a cursor variable points a row which cannot be stored in a two-dimensional PL/SQL table.

537. Difference between NO DATA FOUND and %NOTFOUND?

NO DATA FOUND is an exception raised only for the SELECT....INTO statements when the where clause of the querydoes not match any rows. When the where clause of the explicit cursor does not match any rows the %NOTFOUND attribute is set to TRUE instead.

538. Can you use a commit statement within a database trigger?

No

539. What WHERE CURRENT OF clause does in a cursor?

LOOP
SELECT num_credits INTO v_numcredits FROM classes
WHERE dept=123 and course=101;
UPDATE students
FHKO;;;;;;;;;SET current_credits=current_credits+v_numcredits
WHERE CURRENT OF X;

540. There is a string 120000 12 0 .125 , how you will find the position of the decimal place?

INSTR('120000 12 0 .125',1,'.')
output 13

541. What are different modes of parameters used in functions and procedures?

-IN -OUT -INOUT

542. How you were passing cursor variables in PL/SQL 2.2?

In PL/SQL 2.2 cursor variables cannot be declared in a package.This is because the storage for a cursor variable has to be allocated using Pro*C or OCI with version 2.2, the only means of passing a cursor variable to a PL/SQL block is via bind variable or a procedure parameter.

543. When do you use WHERE clause and when do you use HAVING clause?

HAVING clause is used when you want to specify a condition for a group function and it is written after GROUP BY clause. The WHERE clause is used when you want to specify a condition for columns, single row functions except group functions and it is written before GROUP BY clause if it is used.

544. Difference between procedure and function.?

Functions are named PL/SQL blocks that return a value and can be called with arguments procedure a named block that can be called with parameter. A procedure all is a PL/SQL statement by itself, while a Function call is called as part of an expression.

545. Which is more faster - IN or EXISTS?

EXISTS is more faster than IN because EXISTS returns a Boolean value whereas IN returns a value.

546. What is syntax for dropping a procedure and a function .Are these operations possible?

Drop Procedure procedure_name
Drop Function function_name

547. How will you delete duplicating rows from a base table?

delete from table_name where rowid not in (select max(rowid) from table group by duplicate_values_field_name); or delete duplicate_values_field_name dv from table_name ta where rowid <(select min(rowid) from table_name tb where ta.dv=tb.dv);

548. Difference between database triggers and form triggers?

-Data base trigger(DBT) fires when a DML operation is performed on a data base table. Form trigger(FT) Fires when user presses a key or navigates between fields on the screen
-Can be row level or statement level No distinction between row level and statement level.
-Can manipulate data stored in Oracle tables via SQL Can manipulate data in Oracle tables as well as variables in forms.
-Can be fired from any session executing the triggering DML statements. Can be fired only from the form that define the trigger.
-Can cause other database triggers to fire.Can cause other database triggers to fire, but not other form triggers.

549. What is a cursor for loop?

Cursor For Loop is a loop where oracle implicitly declares a loop variable, the loop index that of the same record type as the cursor's record.

550. How you will avoid duplicating records in a query?

By using DISTINCT

551. What is a view ?

A view is stored procedure based on one or more tables, its a virtual table.

552. What is difference between UNIQUE and PRIMARY KEY constraints?

A table can have only one PRIMARY KEY whereas there can be any number of UNIQUE keys. The columns that compose PK are automatically define NOT NULL, whereas a column that compose a UNIQUE is not automatically defined to be mandatory must also specify the column is NOT NULL.

553. What is use of a cursor variable? How it is defined?

A cursor variable is associated with different statements at run time, which can hold different values at run time. Static cursors can only be associated with one run time query. A cursor variable is reference type (like a pointer in C).
Declaring a cursor variable:
TYPE type_name IS REF CURSOR RETURN return_type type_name is the name of the reference type,return_type is a record type indicating the types of the select list that will eventually be returned by the cursor variable.

554. How do you find the numbert of rows in a Table ?

A bad answer is count them (SELECT COUNT(*) FROM table_name)
A good answer is :-
'By generating SQL to ANALYZE TABLE table_name COUNT STATISTICS by querying Oracle System Catalogues (e.g. USER_TABLES or ALL_TABLES).
The best answer is to refer to the utility which Oracle released which makes it unnecessary to do ANALYZE TABLE for each Table individually.

555. What is the maximum buffer size that can be specified using the DBMS_OUTPUT.ENABLE function?

1,000,00

556. What are cursor attributes?

-%ROWCOUNT
-%NOTFOUND
-%FOUND
-%ISOPEN

557. There is a % sign in one field of a column. What will be the query to find it?

'' Should be used before '%'.

558. What is ON DELETE CASCADE ?

When ON DELETE CASCADE is specified ORACLE maintains referential integrity by automatically removing dependent foreign key values if a referenced primary or unique key value is removed.

559. What is the fastest way of accessing a row in a table ?

Using ROWID.CONSTRAINTS

560. What is difference between TRUNCATE & DELETE ?

TRUNCATE commits after deleting entire table i.e., can not be rolled back. Database triggers do not fire on TRUNCATEDELETE allows the filtered deletion. Deleted records can be rolled back or committed.Database triggers fire on DELETE.

561. What is a transaction ?

Transaction is logical unit between two commits and commit and rollback.

562. What are the advantages of VIEW ?

To protect some of the columns of a table from other users.To hide complexity of a query.To hide complexity of calculations.

563. How will you a activate/deactivate integrity constraints ?

The integrity constraints can be enabled or disabled by ALTER TABLE ENABLE constraint/DISABLE constraint.

564. Where the integrity constraints are stored in Data Dictionary ?

The integrity constraints are stored in USER_CONSTRAINTS.

565. What is the Subquery ?

Sub query is a query whose return values are used in filtering conditions of the main query.

566. How to access the current value and next value from a sequence ? Is it possible to access the current value in a session before accessing next value ?

Sequence name CURRVAL, Sequence name NEXTVAL.It is not possible. Only if you access next value in the session, current value can be accessed.

567. What are the usage of SAVEPOINTS ?value in a session before accessing next value ?

SAVEPOINTS are used to subdivide a transaction into smaller parts. It enables rolling back part of a transaction. Maximum of five save points are allowed.

568. What is ROWID ?in a session before accessing next value ?

ROWID is a pseudo column attached to each row of a table. It is 18 character long, blockno, rownumber are the components of ROWID.

569. Explain Connect by Prior ?in a session before accessing next value ?

Retrieves rows in hierarchical order.e.g. select empno, ename from emp where.

570. How many LONG columns are allowed in a table ? Is it possible to use LONG columns in WHERE clause or ORDER BY ?

Only one LONG columns is allowed. It is not possible to use LONG column in WHERE or ORDER BY clause.

571. What is Referential Integrity ?

Maintaining data integrity through a set of rules that restrict the values of one or more columns of the tables based on the values of primary key or unique key of the referenced table.

572. What is a join ? Explain the different types of joins ?

Join is a query which retrieves related columns or rows from multiple tables.Self Join - Joining the table with itself.Equi Join - Joining two tables by equating two common columns.Non-Equi Join - Joining two tables by equating two common columns.Outer Join - Joining two tables in such a way that query can also retrieve rows that do not have corresponding join value in the other table.

573. If an unique key constraint on DATE column is created, will it validate the rows that are inserted with SYSDATE ?

It won't, Because SYSDATE format contains time attached with it.

574. Explain UNION,MINUS,UNION ALL, INTERSECT ?

INTERSECT returns all distinct rows selected by both queries.MINUS - returns all distinct rows selected by the first query but not by the second.UNION - returns all distinct rows selected by either queryUNION ALL - returns all rows selected by either query, including all duplicates.

575. What is an Integrity Constraint ?

Integrity constraint is a rule that restricts values to a column in a table.

576. Difference between SUBSTR and INSTR ?

INSTR (String1,String2(n,(m)),INSTR returns the position of the mth occurrence of the string 2 instring1. The search begins from nth position of string1.SUBSTR (String1 n,m)SUBSTR returns a character string of size m in string1, starting from nth position of string1.

577. If a View on a single base table is manipulated will the changes be reflected on the base table ?

If changes are made to the tables which are base tables of a view will the changes be reference on the view.

578. What are the pre requisites ?

I. to modify data type of a column ? ii. to add a column with NOT NULL constraint ? To Modify the datatype of a column the column must be empty. to add a column with NOT NULL constrain, the table must be empty.

579. What is a database link ?

Database Link is a named path through which a remote database can be accessed.

580. What are the types of SQL Statement ?

Data Definition Language :
CREATE,ALTER,DROP,TRUNCATE,REVOKE,NO AUDIT & COMMIT.

Data Manipulation Language:
INSERT,UPDATE,DELETE,LOCK

TABLE,EXPLAIN PLAN & SELECT.Transactional Control:
COMMIT & ROLLBACKSession Control: ALTERSESSION & SET

ROLESystem Control :
ALTER SYSTEM.

581. What is CYCLE/NO CYCLE in a Sequence ?

CYCLE specifies that the sequence continues to generate values after reaching either maximum or minimum value. After pan ascending sequence reaches its maximum value, it generates its minimum value. After a descending sequence reaches its minimum, it generates its maximum.NO CYCLE specifies that the sequence cannot generate more values after reaching its maximum or minimum value.

582. What is correlated sub-query ?

Correlated sub query is a sub query which has reference to the main query.

583. What are the data types allowed in a table ?

CHAR,VARCHAR2,NUMBER,DATE,RAW,LONG and LONG RAW.

584. What is difference between CHAR and VARCHAR2 ? What is the maximum SIZE allowed for each type ?

CHAR pads blank spaces to the maximum length. VARCHAR2 does not pad blank spaces. For CHAR it is 255 and 2000 for VARCHAR2.

585. Can a view be updated/inserted/deleted? If Yes under what conditions ?

A View can be updated/deleted/inserted if it has only one base table if the view is based on columns from one or more tables then insert, update and delete is not possible.

586. What are the different types of Coordinations of the Master with the Detail block?

POPULATE_GROUP(function)
POPULATE_GROUP_WITH_QUERY(function)
SET_GROUP_CHAR_CELL(procedure)
SET_GROUPCELL(procedure)
SET_GROUP_NUMBER_CELL(procedure)

587. Use the ADD_GROUP_COLUMN function to add a column to a record group that was created at design time? I) TRUE II)FALSE

II) FALSE

588. Use the ADD_GROUP_ROW procedure to add a row to a static record group? I) TRUE II)FALSE

I) FALSE

589. maxvalue.sql Select the Nth Highest value from a table?

select level, max('col_name') from my_table where level = '&n' connect by prior ('col_name') > 'col_name')
group by level;
Example:
Given a table called emp with the following columns:
-- id number
-- name varchar2(20)
-- sal number
--
-- For the second highest salary:
-- select level, max(sal) from emp
-- where level=2
-- connect by prior sal > sal
-- group by level

590. Find out nth highest salary from emp table?

SELECT DISTINCT (a.sal) FROM EMP A WHERE &N = (SELECT COUNT (DISTINCT (b.sal)) FROM EMP B WHERE a.sal<=b.sal);
For Eg:-
Enter value for n: 2
SAL
---------
3700

591. Suppose a customer table is having different columns like customer no, payments.What will be the query to select top three max payments?

SELECT customer_no, payments from customer C1
WHERE 3<=(SELECT COUNT(*) from customer C2
WHERE C1.payment <= C2.payment)

592. How you will avoid your query from using indexes?

SELECT * FROM emp
Where emp_no+' '=12345;
i.e you have to concatenate the column name with space within codes in the where condition.
SELECT /*+ FULL(a) */ ename, emp_no from emp
where emp_no=1234;
i.e using HINTS

593. What utility is used to create a physical backup?

Either rman or alter tablespace begin backup will do..

Oracel Interview Question Only (1)

  • What is meant by Scrollable cursor?
  • The use of HAVING , WHERE and GROUPBY in one SQL?
  • What is meant by SORTING and GROUPING?
  • What are the different types of OUTER JOINS?
  • Explain the UNION operation in SQL . What is meant by OUTER JOIN?
  • What is a Non- corelated subquery?
  • What is a corelated subquery?
  • What do you know about subqueries?
  • What is the theory behind the JOIN statement in DB2?
  • What are the different types of SQL?
  • How to drop the column in a table?
  • How to drop the index?
  • What are the different tablespaces in database?
  • What is the maximum number of triggers, can apply to a single table?
  • What is the output of SIGN function?
  • What are the more common pseudo-columns?
  • Other way to replace query result null value with a text?
  • What are PL/SQL Cursor Exceptions?
  • Any three PL/SQL Exceptions?
  • Any three PL/SQL Exceptions?
  • Which date function returns number value?
  • Display Odd/ Even number of records?
  • Display the number value in Words?
  • To view installed Oracle version information?
  • Find out nth highest salary from emp table?
  • Implicit Cursor attributes?
  • Explicit Cursor attributes?
  • Display the records between two range I know the nvl function only allows the same data type(ie. number or char or date Nvl(comm, 0)), if commission is null then the text Not Applicable want to display, instead of blank space. How do I write the query???
  • How do I display row number with records?
  • How do I eliminate the duplicate rows?
  • If a view on a single base table is manipulated will the changes be reflected on the base table?
  • If Yes - under what conditions?
  • Can a view be updated/inserted/deleted?
  • What are the advantages of VIEW?
  • What is CYCLE/NO CYCLE in a Sequence?
  • Is it possible to access the current value in a session before accessing next value?
  • How to access the current value and next value from a sequence?
  • What is a database link?
  • If unique key constraint on DATE column is created, will it validate the rows that are inserted with SYSDATE?
  • How will you activate/deactivate integrity constraints?
  • Where the integrity constraints are stored in data dictionary?
  • What are the pre-requisites to modify datatype of a column and to add a column with NOT NULL constraint?
  • Is it possible to use LONG columns in WHERE clause or ORDER BY?
  • How many LONG columns are allowed in a table?
  • What is the maximum SIZE allowed for each type?
  • What is difference between CHAR and VARCHAR2?
  • What are the data types allowed in a table?
  • What is ON DELETE CASCADE?
  • What is the usage of SAVEPOINTS?
  • What is referential integrity constraint?
  • What is an integrity constraint?
  • What is the fastest way of accessing a row in a table?
  • What is ROWID?
  • Explain UNION, MINUS, UNION ALL and INTERSECT?
  • Difference between SUBSTR and INSTR?
  • Explain CONNECT BY PRIOR?
  • What is correlated sub-query?
  • What is the sub-query?
  • Explain the different types of joins?
  • What is a join?
  • What is difference between TRUNCATE & DELETE?
  • What is a transaction?
  • What are the types of SQL statement?
  • Which datatype is used for storing graphics and images?
  • LONG RAW data type is used for storing BLOB's (binary large objects).?
  • What is a pseudo column. Give some examples?
  • It is a column that is not an actual column in the table?
  • eg USER, UID, SYSDATE, ROWNUM, ROWID, NULL, AND LEVEL.?
  • What are the differences you have seen while installing Oracle on NT and Unix platform?
  • What utility is used to create a logical backup? export

Oracel Interview Question Only (2)

  • What are the differences between database designing and database modeling?
  • If the large table contains thousands of records and the application is accessing 35% of the table which method to use: index searching or full table scan?
  • In which situation whether peak time or off peak time you will execute the ANALYZE TABLE command. Why?
  • How to check to memory gap once the SGA is started in Restricted mode?
  • All the users are complaining that their application is hanging. How you will resolve this situation in OLTP?
  • If the SQL * Plus hangs for a long time, what is the reason?
  • Shall we create procedures to fetch more than one record?
  • How do you increase the performance of %LIKE operator?
  • You are regularly changing the package body part. How will you create or what will you do before creating that package?
  • How can you see the source code of the package?
  • Dual table explain. Is any data internally storing in dual table. Lot of users are accessing select sysdate from dual and they getting some millisecond differences. If we execute SELECT SYSDATE FROM EMP; what error will we get. Why?
  • In exception handling we have some NOT_FOUND and OTHERS. In inner layer we have some NOT_FOUND and OTHERS. While executing which one whether outer layer or inner layer will check first?
  • What is mutated trigger, is it the problem of locks. In single user mode we got mutated error, as a DBA how you will resolve it?
  • Schema A has some objects and created one procedure and granted to Schema B. Schema B has the same objects like schema A. Schema B executed the procedure like inserting some records. In this case where the data will be stored whether in Schema A or Schema B?
  • What is bulk SQL?
  • How to do the scheduled task/jobs in Unix platform?
  • If the entire disk is corrupted how will you and what are the steps to recover the database?
  • How will you monitor rollback segment status?
  • List the sequence of events when a large transaction that exceeds beyond its optimal value when an entry wraps and causes the rollback segment to expand into another extend?
  • What is redo log file mirroring?
  • How can we plan storage for very large tables
  • When will be a segment released ?
  • What are disadvantages of having raw devices?
  • List the factors that can affect the accuracy of the estima?
  • What is the difference between $$DATE$$ & $$DBDATE$$$$DBDATE$$ retrieves the current database date$$date$$ retrieves the current operating system
  • How to prevent unauthorized use of privileges granted to a Role ?
  • What is a deadlock and Explain?
  • What are the basic element of base configuration of an Oracle database?
  • What is an index and How it is implemented in Oracle database?
  • What is the use of redo log information?
  • What is a schema?
  • What is Parallel Server?
  • What is a database instance and Explain?
  • What is a datafile?
  • What is a temporary segment?
  • What are the uses of rollback segment


Oracle Interview Questions

  1. Explain the difference between a hot backup and a cold backup and the benefits associated with each.
  2. A hot backup is basically taking a backup of the database while it is still up and running and it must be in archive log mode. A cold backup is taking a backup of the database while it is shut down and does not require being in archive log mode. The benefit of taking a hot backup is that the database is still available for use while the backup is occurring and you can recover the database to any ball in time. The benefit of taking a cold backup is that it is typically easier to administer the backup and recovery process. In addition, since you are taking cold backups the database does not require being in archive log mode and thus there will be a slight performance gain as the database is not cutting archive logs to disk.

  3. You have just had to restore from backup and do not have any control files. How would you go about bringing up this database?
  4. I would create a text based backup control file, stipulating where on disk all the data files where and then issue the recover command with the using backup control file clause.

  5. How do you switch from an init.ora file to a spfile?
  6. Issue the create spfile from pfile command.

  7. Explain the difference between a data block, an extent and a segment.
  8. A data block is the smallest unit of logical storage for a database object. As objects grow they take chunks of additional storage that are composed of contiguous data blocks. These groupings of contiguous data blocks are called extents. All the extents that an object takes when grouped together are considered the segment of the database object.

  9. Give two examples of how you might determine the structure of the table DEPT.
  10. Use the describe command or use the dbms_metadata.get_ddl package.

  11. Where would you look for errors from the database engine?
  12. In the alert log.

  13. Compare and contrast TRUNCATE and DELETE for a table.
  14. Both the truncate and delete command have the desired outcome of getting rid of all the rows in a table. The difference between the two is that the truncate command is a DDL operation and just moves the high water mark and produces a now rollback. The delete command, on the other hand, is a DML operation, which will produce a rollback and thus take longer to complete.

  15. Give the reasoning behind using an index.
  16. Faster access to data blocks in a table.

  17. Give the two types of tables involved in producing a star schema and the type of data they hold.
  18. Fact tables and dimension tables. A fact table contains measurements while dimension tables will contain data that will help describe the fact tables.

  19. What type of index should you use on a fact table?
  20. A Bitmap index.

  21. Give two examples of referential integrity constraints.
  22. A primary key and a foreign key.

  23. A table is classified as a parent table and you want to drop and re-create it. How would you do this without affecting the children tables?
  24. Disable the foreign key constraint to the parent, drop the table, re-create the table, enable the foreign key constraint.

  25. Explain the difference between ARCHIVELOG mode and NOARCHIVELOG mode and the benefits and disadvantages to each.
  26. ARCHIVELOG mode is a mode that you can put the database in for creating a backup of all transactions that have occurred in the database so that you can recover to any ball in time. NOARCHIVELOG mode is basically the absence of ARCHIVELOG mode and has the disadvantage of not being able to recover to any ball in time. NOARCHIVELOG mode does have the advantage of not having to write transactions to an archive log and thus increases the performance of the database slightly.

  27. What command would you use to create a backup control file?
  28. Alter database backup control file to trace.

  29. Give the stages of instance startup to a usable state where normal users may access it.
  30. STARTUP NOMOUNT - Instance startup

    STARTUP MOUNT - The database is mounted

    STARTUP OPEN - The database is opened

  31. What column differentiates the V$ views to the GV$ views and how?
  32. The INST_ID column which indicates the instance in a RAC environment the information came from.

  33. How would you go about generating an EXPLAIN plan?
  34. Create a plan table with utlxplan.sql.

    Use the explain plan set statement_id = 'tst1' into plan_table for a SQL statement

    Look at the explain plan with utlxplp.sql or utlxpls.sql

  35. How would you go about increasing the buffer cache hit ratio?
  36. Use the buffer cache advisory over a given workload and then query the v$db_cache_advice table. If a change was necessary then I would use the alter system set db_cache_size command.

  37. Explain an ORA-01555
  38. You get this error when you get a snapshot too old within rollback. It can usually be solved by increasing the undo retention or increasing the size of rollbacks. You should also look at the logic involved in the application getting the error message.

  39. Explain the difference between $ORACLE_HOME and $ORACLE_BASE.

ORACLE_BASE is the root directory for oracle. ORACLE_HOME located beneath ORACLE_BASE is where the oracle products reside.

  1. Describe the difference between a procedure, function and anonymous pl/sql block.
  2. Candidate should mention use of DECLARE statement, a function must return a value while a procedure doesn?t have to.

     

  3. What is a mutating table error and how can you get around it?
  4. This happens with triggers. It occurs because the trigger is trying to update a row it is currently using. The usual fix involves either use of views or temporary tables so the database is selecting from one while updating the other.

     

  5. Describe the use of %ROWTYPE and %TYPE in PL/SQL
  6. Expected answer: %ROWTYPE allows you to associate a variable with an entire table row. The %TYPE associates a variable with a single column type.

     

  7. 4What packages (if any) has Oracle provided for use by developers?
  8. Expected answer: Oracle provides the DBMS_ series of packages. There are many which developers should be aware of such as DBMS_SQL, DBMS_PIPE, DBMS_TRANSACTION, DBMS_LOCK, DBMS_ALERT, DBMS_OUTPUT, DBMS_JOB, DBMS_UTILITY, DBMS_DDL, UTL_FILE. If they can mention a few of these and describe how they used them, even better. If they include the SQL routines provided by Oracle, great, but not really what was asked.

     

  9. Describe the use of PL/SQL tables
  10. Expected answer: PL/SQL tables are scalar arrays that can be referenced by a binary integer. They can be used to hold values for use in later queries or calculations. In Oracle 8 they will be able to be of the %ROWTYPE designation, or RECORD.

     

  11. When is a declare statement needed ?
  12. The DECLARE statement is used in PL/SQL anonymous blocks such as with stand alone, non-stored PL/SQL procedures. It must come first in a PL/SQL stand alone file if it is used.

     

  13. In what order should a open/fetch/loop set of commands in a PL/SQL block be implemented if you use the %NOTFOUND cursor variable in the exit when statement? Why?
  14. Expected answer: OPEN then FETCH then LOOP followed by the exit when. If not specified in this order will result in the final return being done twice because of the way the %NOTFOUND is handled by PL/SQL.

     

  15. What are SQLCODE and SQLERRM and why are they important for PL/SQL developers?
  16. Expected answer: SQLCODE returns the value of the error number for the last error encountered. The SQLERRM returns the actual error message for the last error encountered. They can be used in exception handling to report, or, store in an error log table, the error that occurred in the code. These are especially useful for the WHEN OTHERS exception.

     

  17. How can you find within a PL/SQL block, if a cursor is open?
  18. Expected answer: Use the %ISOPEN cursor status variable.

     

  19. How can you generate debugging output from PL/SQL?
  20. Expected answer: Use the DBMS_OUTPUT package. Another possible method is to just use the SHOW ERROR command, but this only shows errors. The DBMS_OUTPUT package can be used to show intermediate results from loops and the status of variables as the procedure is executed. The new package UTL_FILE can also be used.

     

  21. What are the types of triggers?

Expected Answer: There are 12 types of triggers in PL/SQL that consist of combinations of the BEFORE, AFTER, ROW, TABLE, INSERT, UPDATE, DELETE and ALL key words:

BEFORE ALL ROW INSERT

AFTER ALL ROW INSERT

BEFORE INSERT

AFTER INSERT etc.

  1. Give one method for transferring a table from one schema to another:
  2. There are several possible methods, export-import, CREATE TABLE... AS SELECT, or COPY.

     

  3. What is the purpose of the IMPORT option IGNORE? What is it?s default setting
  4. The IMPORT IGNORE option tells import to ignore "already exists" errors. If it is not specified the tables that already exist will be skipped. If it is specified, the error is ignored and the tables data will be inserted. The default value is N.

     

  5. You have a rollback segment in a version 7.2 database that has expanded beyond optimal, how can it be restored to optimal
  6. Use the ALTER TABLESPACE ..... SHRINK command.

     

  7. If the DEFAULT and TEMPORARY tablespace clauses are left out of a CREATE USER command what happens? Is this bad or good? Why
  8. The user is assigned the SYSTEM tablespace as a default and temporary tablespace. This is bad because it causes user objects and temporary segments to be placed into the SYSTEM tablespace resulting in fragmentation and improper table placement (only data dictionary objects and the system rollback segment should be in SYSTEM).

     

  9. What are some of the Oracle provided packages that DBAs should be aware of
  10. Oracle provides a number of packages in the form of the DBMS_ packages owned by the SYS user. The packages used by DBAs may include: DBMS_SHARED_POOL, DBMS_UTILITY, DBMS_SQL, DBMS_DDL, DBMS_SESSION, DBMS_OUTPUT and DBMS_SNAPSHOT. They may also try to answer with the UTL*.SQL or CAT*.SQL series of SQL procedures. These can be viewed as extra credit but aren?t part of the answer.

     

  11. What happens if the constraint name is left out of a constraint clause
  12. The Oracle system will use the default name of SYS_Cxxxx where xxxx is a system generated number. This is bad since it makes tracking which table the constraint belongs to or what the constraint does harder.

     

  13. What happens if a tablespace clause is left off of a primary key constraint clause
  14. This results in the index that is automatically generated being placed in then users default tablespace. Since this will usually be the same tablespace as the table is being created in, this can cause serious performance problems.

     

  15. What is the proper method for disabling and re-enabling a primary key constraint
  16. You use the ALTER TABLE command for both. However, for the enable clause you must specify the USING INDEX and TABLESPACE clause for primary keys.

     

  17. What happens if a primary key constraint is disabled and then enabled without fully specifying the index clause
  18. The index is created in the user?s default tablespace and all sizing information is lost. Oracle doesn?t store this information as a part of the constraint definition, but only as part of the index definition, when the constraint was disabled the index was dropped and the information is gone.

     

  19. (On UNIX) When should more than one DB writer process be used? How many should be used
  20. If the UNIX system being used is capable of asynchronous IO then only one is required, if the system is not capable of asynchronous IO then up to twice the number of disks used by Oracle number of DB writers should be specified by use of the db_writers initialization parameter.

     

  21. You are using hot backup without being in archivelog mode, can you recover in the event of a failure? Why or why not
  22. You can?t use hot backup without being in archivelog mode. So no, you couldn?t recover.

     

  23. What causes the "snapshot too old" error? How can this be prevented or mitigated
  24. This is caused by large or long running transactions that have either wrapped onto their own rollback space or have had another transaction write on part of their rollback space. This can be prevented or mitigated by breaking the transaction into a set of smaller transactions or increasing the size of the rollback segments and their extents.

     

    13. How can you tell if a database object is invalid

    By checking the status column of the DBA_, ALL_ or USER_OBJECTS views, depending upon whether you own or only have permission on the view or are using a DBA account.

     

  25. A user is getting an ORA-00942 error yet you know you have granted them permission on the table, what else should you check
  26. You need to check that the user has specified the full name of the object (select empid from scott.emp; instead of select empid from emp;) or has a synonym that balls to the object (create synonym emp for scott.emp;)

     

  27. A developer is trying to create a view and the database won?t let him. He has the "DEVELOPER" role which has the "CREATE VIEW" system privilege and SELECT grants on the tables he is using, what is the problem
  28. You need to verify the developer has direct grants on all tables used in the view. You can?t create a stored object with grants given through views.

     

  29. If you have an example table, what is the best way to get sizing data for the production table implementation
  30. The best way is to analyze the table and then use the data provided in the DBA_TABLES view to get the average row length and other pertinent data for the calculation. The quick and dirty way is to look at the number of blocks the table is actually using and ratio the number of rows in the table to its number of blocks against the number of expected rows.

     

  31. How can you find out how many users are currently logged into the database? How can you find their operating system id
  32. There are several ways. One is to look at the v$session or v$process views. Another way is to check the current_logins parameter in the v$sysstat view. Another if you are on UNIX is to do a "ps -ef|grep oracle|wc -l? command, but this only works against a single instance installation.

     

  33. A user selects from a sequence and gets back two values, his select is:
  34. SELECT pk_seq.nextval FROM dual;What is the problem

    Somehow two values have been inserted into the dual table. This table is a single row, single column table that should only have one value in it.

     

  35. How can you determine if an index needs to be dropped and rebuilt

Run the ANALYZE INDEX command on the index to validate its structure and then calculate the ratio of LF_BLK_LEN/LF_BLK_LEN+BR_BLK_LEN and if it isn?t near 1.0 (i.e. greater than 0.7 or so) then the index should be rebuilt. Or if the ratio

BR_BLK_LEN/ LF_BLK_LEN+BR_BLK_LEN is nearing 0.3

  1. How can variables be passed to a SQL routine
  2. By use of the & symbol. For passing in variables the numbers 1-8 can be used (&1, &2,...,&8) to pass the values after the command into the SQLPLUS session. To be prompted for a specific variable, place the ampersanded variable in the code itself:

    "select * from dba_tables where owner=&owner_name;" . Use of double ampersands tells SQLPLUS to resubstitute the value for each subsequent use of the variable, a single ampersand will cause a reprompt for the value unless an ACCEPT statement is used to get the value from the user.

     

  3. You want to include a carriage return/linefeed in your output from a SQL script, how can you do this
  4. The best method is to use the CHR() function (CHR(10) is a return/linefeed) and the concatenation function "||". Another method, although it is hard to document and isn?t always portable is to use the return/linefeed as a part of a quoted string.

     

  5. How can you call a PL/SQL procedure from SQL
  6. By use of the EXECUTE (short form EXEC) command.

     

  7. How do you execute a host operating system command from within SQL
  8. By use of the exclamation ball "!" (in UNIX and some other OS) or the HOST (HO) command.

     

  9. You want to use SQL to build SQL, what is this called and give an example
  10. This is called dynamic SQL. An example would be:

    set lines 90 pages 0 termout off feedback off verify off

    spool drop_all.sql

    select ?drop user ?||username||? cascade;? from dba_users

    where username not in ("SYS?,?SYSTEM?);

    spool off

    Essentially you are looking to see that they know to include a command (in this case DROP USER...CASCADE;) and that you need to concatenate using the ?||? the values selected from the database.

     

  11. What SQLPlus command is used to format output from a select
  12. This is best done with the COLUMN command.

     

  13. You want to group the following set of select returns, what can you group on
  14. Max(sum_of_cost), min(sum_of_cost), count(item_no), item_no

    The only column that can be grouped on is the "item_no" column, the rest have aggregate functions associated with them.

     

  15. What special Oracle feature allows you to specify how the cost based system treats a SQL statement
  16. The COST based system allows the use of HINTs to control the optimizer path selection. If they can give some example hints such as FIRST ROWS, ALL ROWS, USING INDEX, STAR, even better.

     

  17. You want to determine the location of identical rows in a table before attempting to place a unique index on the table, how can this be done
  18. Oracle tables always have one guaranteed unique column, the rowid column. If you use a min/max function against your rowid and then select against the proposed primary key you can squeeze out the rowids of the duplicate rows pretty quick. For example:

    select rowid from emp e

    where e.rowid > (select min(x.rowid)

    from emp x

    where x.emp_no = e.emp_no);

    In the situation where multiple columns make up the proposed key, they must all be used in the where clause.

     

  19. What is a Cartesian product
  20. A Cartesian product is the result of an unrestricted join of two or more tables. The result set of a three table Cartesian product will have x * y * z number of rows where x, y, z correspond to the number of rows in each table involved in the join.

     

    11. You are joining a local and a remote table, the network manager complains about the traffic involved, how can you reduce the network traffic

    Push the processing of the remote data to the remote instance by using a view to pre-select the information for the join. This will result in only the data required for the join being sent across.

     

  21. What is the default ordering of an ORDER BY clause in a SELECT statement
  22. Ascending

     

  23. What is tkprof and how is it used
  24. The tkprof tool is a tuning tool used to determine cpu and execution times for SQL statements. You use it by first setting timed_statistics to true in the initialization file and then turning on tracing for either the entire database via the sql_trace parameter or for the session using the ALTER SESSION command. Once the trace file is generated you run the tkprof tool against the trace file and then look at the output from the tkprof tool. This can also be used to generate explain plan output.

     

  25. What is explain plan and how is it used
  26. The EXPLAIN PLAN command is a tool to tune SQL statements. To use it you must have an explain_table generated in the user you are running the explain plan for. This is created using the utlxplan.sql script. Once the explain plan table exists you run the explain plan command giving as its argument the SQL statement to be explained. The explain_plan table is then queried to see the execution plan of the statement. Explain plans can also be run using tkprof.

     

  27. How do you set the number of lines on a page of output? The width
  28. The SET command in SQLPLUS is used to control the number of lines generated per page and the width of those lines, for example SET PAGESIZE 60 LINESIZE 80 will generate reports that are 60 lines long with a line width of 80 characters. The PAGESIZE and LINESIZE options can be shortened to PAGES and LINES.

     

  29. How do you prevent output from coming to the screen
  30. The SET option TERMOUT controls output to the screen. Setting TERMOUT OFF turns off screen output. This option can be shortened to TERM.

     

  31. How do you prevent Oracle from giving you informational messages during and after a SQL statement execution
  32. The SET options FEEDBACK and VERIFY can be set to OFF.

     

  33. How do you generate file output from SQL

By use of the SPOOL command

  1. A tablespace has a table with 30 extents in it. Is this bad? Why or why not.
  2. Multiple extents in and of themselves aren?t bad. However if you also have chained rows this can hurt performance.

     

  3. How do you set up tablespaces during an Oracle installation?
  4. You should always attempt to use the Oracle Flexible Architecture standard or another partitioning scheme to ensure proper separation of SYSTEM, ROLLBACK, REDO LOG, DATA, TEMPORARY and INDEX segments.

     

  5. You see multiple fragments in the SYSTEM tablespace, what should you check first?
  6. Ensure that users don?t have the SYSTEM tablespace as their TEMPORARY or DEFAULT tablespace assignment by checking the DBA_USERS view.

     

  7. What are some indications that you need to increase the SHARED_POOL_SIZE parameter?
  8. Poor data dictionary or library cache hit ratios, getting error ORA-04031. Another indication is steadily decreasing performance with all other tuning parameters the same.

     

  9. What is the general guideline for sizing db_block_size and db_multi_block_read for an application that does many full table scans?
  10. Oracle almost always reads in 64k chunks. The two should have a product equal to 64 or a multiple of 64.

     

  11. What is the fastest query method for a table
  12. Fetch by rowid

     

  13. Explain the use of TKPROF? What initialization parameter should be turned on to get full TKPROF output?
  14. The tkprof tool is a tuning tool used to determine cpu and execution times for SQL statements. You use it by first setting timed_statistics to true in the initialization file and then turning on tracing for either the entire database via the sql_trace parameter or for the session using the ALTER SESSION command. Once the trace file is generated you run the tkprof tool against the trace file and then look at the output from the tkprof tool. This can also be used to generate explain plan output.

     

  15. When looking at v$sysstat you see that sorts (disk) is high. Is this bad or good? If bad -How do you correct it?
  16. If you get excessive disk sorts this is bad. This indicates you need to tune the sort area parameters in the initialization files. The major sort are parameter is the SORT_AREA_SIZe parameter.

     

  17. When should you increase copy latches? What parameters control copy latches
  18. When you get excessive contention for the copy latches as shown by the "redo copy" latch hit ratio. You can increase copy latches via the initialization parameter LOG_SIMULTANEOUS_COPIES to twice the number of CPUs on your system.

     

  19. Where can you get a list of all initialization parameters for your instance? How about an indication if they are default settings or have been changed
  20. You can look in the init.ora file for an indication of manually set parameters. For all parameters, their value and whether or not the current value is the default value, look in the v$parameter view.

     

  21. Describe hit ratio as it pertains to the database buffers. What is the difference between instantaneous and cumulative hit ratio and which should be used for tuning
  22. The hit ratio is a measure of how many times the database was able to read a value from the buffers verses how many times it had to re-read a data value from the disks. A value greater than 80-90% is good, less could indicate problems. If you simply take the ratio of existing parameters this will be a cumulative value since the database started. If you do a comparison between pairs of readings based on some arbitrary time span, this is the instantaneous ratio for that time span. Generally speaking an instantaneous reading gives more valuable data since it will tell you what your instance is doing for the time it was generated over.

     

  23. Discuss row chaining, how does it happen? How can you reduce it? How do you correct it
  24. Row chaining occurs when a VARCHAR2 value is updated and the length of the new value is longer than the old value and won?t fit in the remaining block space. This results in the row chaining to another block. It can be reduced by setting the storage parameters on the table to appropriate values. It can be corrected by export and import of the effected table.

     

  25. When looking at the estat events report you see that you are getting busy buffer waits. Is this bad? How can you find what is causing it
  26. Buffer busy waits could indicate contention in redo, rollback or data blocks. You need to check the v$waitstat view to see what areas are causing the problem. The value of the "count" column tells where the problem is, the "class" column tells you with what. UNDO is rollback segments, DATA is data base buffers.

     

  27. If you see contention for library caches how can you fix it
  28. Increase the size of the shared pool.

     

  29. If you see statistics that deal with "undo" what are they really talking about
  30. Rollback segments and associated structures.

     

  31. If a tablespace has a default pctincrease of zero what will this cause (in relationship to the smon process)
  32. The SMON process won?t automatically coalesce its free space fragments.

     

  33. If a tablespace shows excessive fragmentation what are some methods to defragment the tablespace? (7.1,7.2 and 7.3 only)
  34. In Oracle 7.0 to 7.2 The use of the 'alter session set events 'immediate trace name coalesce level ts#';? command is the easiest way to defragment contiguous free space fragmentation. The ts# parameter corresponds to the ts# value found in the ts$ SYS table. In version 7.3 the ?alter tablespace coalesce;? is best. If the free space isn?t contiguous then export, drop and import of the tablespace contents may be the only way to reclaim non-contiguous free space.

     

  35. How can you tell if a tablespace has excessive fragmentation
  36. If a select against the dba_free_space table shows that the count of a tablespaces extents is greater than the count of its data files, then it is fragmented.

     

  37. You see the following on a status report:
  38. redo log space requests 23

    redo log space wait time 0

    Is this something to worry about? What if redo log space wait time is high? How can you fix this

    Since the wait time is zero, no. If the wait time was high it might indicate a need for more or larger redo logs.

     

  39. What can cause a high value for recursive calls? How can this be fixed
  40. A high value for recursive calls is cause by improper cursor usage, excessive dynamic space management actions, and or excessive statement re-parses. You need to determine the cause and correct it By either relinking applications to hold cursors, use proper space management techniques (proper storage and sizing) or ensure repeat queries are placed in packages for proper reuse.

     

  41. If you see a pin hit ratio of less than 0.8 in the estat library cache report is this a problem? If so, how do you fix it
  42. This indicate that the shared pool may be too small. Increase the shared pool size.

     

  43. If you see the value for reloads is high in the estat library cache report is this a matter for concern
  44. Yes, you should strive for zero reloads if possible. If you see excessive reloads then increase the size of the shared pool.

     

  45. You look at the dba_rollback_segs view and see that there is a large number of shrinks and they are of relatively small size, is this a problem? How can it be fixed if it is a problem
  46. A large number of small shrinks indicates a need to increase the size of the rollback segment extents. Ideally you should have no shrinks or a small number of large shrinks. To fix this just increase the size of the extents and adjust optimal accordingly.

     

  47. You look at the dba_rollback_segs view and see that you have a large number of wraps is this a problem
  48. A large number of wraps indicates that your extent size for your rollback segments are probably too small. Increase the size of your extents to reduce the number of wraps. You can look at the average transaction size in the same view to get the information on transaction size.

     

  49. In a system with an average of 40 concurrent users you get the following from a query on rollback extents:
  50. ROLLBACK CUR EXTENTS

    --------------------- --------------------------

    R01 11

    R02 8

    R03 12

    R04 9

    SYSTEM 4

  51. You have room for each to grow by 20 more extents each. Is there a problem? Should you take any action
  52. No there is not a problem. You have 40 extents showing and an average of 40 concurrent users. Since there is plenty of room to grow no action is needed.

     

  53. You see multiple extents in the temporary tablespace. Is this a problem
  54. As long as they are all the same size this isn?t a problem. In fact, it can even improve performance since Oracle won?t have to create a new extent when a user needs one.

     

  55. Define OFA.
  56. OFA stands for Optimal Flexible Architecture. It is a method of placing directories and files in an Oracle system so that you get the maximum flexibility for future tuning and file placement.

     

  57. How do you set up your tablespace on installation
  58. The answer here should show an understanding of separation of redo and rollback, data and indexes and isolation os SYSTEM tables from other tables. An example would be to specify that at least 7 disks should be used for an Oracle installation so that you can place SYSTEM tablespace on one, redo logs on two (mirrored redo logs) the TEMPORARY tablespace on another, ROLLBACK tablespace on another and still have two for DATA and INDEXES. They should indicate how they will handle archive logs and exports as well. As long as they have a logical plan for combining or further separation more or less disks can be specified.

     

  59. What should be done prior to installing Oracle (for the OS and the disks)
  60. adjust kernel parameters or OS tuning parameters in accordance with installation guide. Be sure enough contiguous disk space is available.

     

  61. You have installed Oracle and you are now setting up the actual instance. You have been waiting an hour for the initialization script to finish, what should you check first to determine if there is a problem
  62. Check to make sure that the archiver isn?t stuck. If archive logging is turned on during install a large number of logs will be created. This can fill up your archive log destination causing Oracle to stop to wait for more space.

     

  63. When configuring SQLNET on the server what files must be set up
  64. INITIALIZATION file, TNSNAMES.ORA file, SQLNET.ORA file

     

  65. When configuring SQLNET on the client what files need to be set up
  66. SQLNET.ORA, TNSNAMES.ORA

     

  67. What must be installed with ODBC on the client in order for it to work with Oracle
  68. SQLNET and PROTOCOL (for example: TCPIP adapter) layers of the transport programs.

     

  69. You have just started a new instance with a large SGA on a busy existing server. Performance is terrible, what should you check for
  70. The first thing to check with a large SGA is that it isn?t being swapped out.

     

  71. What OS user should be used for the first part of an Oracle installation (on UNIX)
  72. You must use root first.

     

  73. When should the default values for Oracle initialization parameters be used as is
  74. Never

     

  75. How many control files should you have? Where should they be located
  76. At least 2 on separate disk spindles. Be sure they say on separate disks, not just file systems.

     

  77. How many redo logs should you have and how should they be configured for maximum recoverability
  78. You should have at least three groups of two redo logs with the two logs each on a separate disk spindle (mirrored by Oracle). The redo logs should not be on raw devices on UNIX if it can be avoided.

     

  79. You have a simple application with no "hot" tables (i.e. uniform IO and access requirements). How many disks should you have assuming standard layout for SYSTEM, USER, TEMP and ROLLBACK tablespaces
  80. At least 7, see disk configuration answer above.

     

  81. Describe third normal form
  82. Something like: In third normal form all attributes in an entity are related to the primary key and only to the primary key

     

  83. Is the following statement true or false:
  84. "All relational databases must be in third normal form"

    False. While 3NF is good for logical design most databases, if they have more than just a few tables, will not perform well using full 3NF. Usually some entities will be denormalized in the logical to physical transfer process.

     

  85. What is an ERD
  86. An ERD is an Entity-Relationship-Diagram. It is used to show the entities and relationships for a database logical model.

     

  87. Why are recursive relationships bad? How do you resolve them
  88. A recursive relationship (one where a table relates to itself) is bad when it is a hard relationship (i.e. neither side is a "may" both are "must") as this can result in it not being possible to put in a top or perhaps a bottom of the table (for example in the EMPLOYEE table you couldn?t put in the PRESIDENT of the company because he has no boss, or the junior janitor because he has no subordinates). These type of relationships are usually resolved by adding a small intersection entity.

     

  89. What does a hard one-to-one relationship mean (one where the relationship on both ends is "must")
  90. Expected answer: This means the two entities should probably be made into one entity.

     

  91. How should a many-to-many relationship be handled
  92. By adding an intersection entity table

     

  93. What is an artificial (derived) primary key? When should an artificial (or derived) primary key be used
  94. A derived key comes from a sequence. Usually it is used when a concatenated key becomes too cumbersome to use as a foreign key.

     

  95. When should you consider denormalization
  96. Whenever performance analysis indicates it would be beneficial to do so without compromising data integrity.

     

  97. How can you determine if an Oracle instance is up from the operating system level
  98. There are several base Oracle processes that will be running on multi-user operating systems, these will be smon, pmon, dbwr and lgwr. Any answer that has them using their operating system process showing feature to check for these is acceptable. For example, on UNIX a ps -ef|grep dbwr will show what instances are up.

     

  99. Users from the PC clients are getting messages indicating :
  100. ORA-06114: (Cnct err, can't get err txt. See Servr Msgs & Codes Manual)

     

    What could the problem be

    The instance name is probably incorrect in their connection string.

     

  101. Users from the PC clients are getting the following error stack:
  102. ERROR: ORA-01034: ORACLE not available

    ORA-07318: smsget: open error when opening sgadef.dbf file.

    HP-UX Error: 2: No such file or directory

     

    What is the probable cause

    The Oracle instance is shutdown that they are trying to access, restart the instance.

     

  103. How can you determine if the SQLNET process is running for SQLNET V1? How about V2
  104. For SQLNET V1 check for the existence of the orasrv process. You can use the command "tcpctl status" to get a full status of the V1 TCPIP server, other protocols have similar command formats. For SQLNET V2 check for the presence of the LISTENER process(s) or you can issue the command "lsnrctl status".

     

  105. What file will give you Oracle instance status information? Where is it located
  106. The alert.ora log. It is located in the directory specified by the background_dump_dest parameter in the v$parameter table.

     

  107. Users aren?t being allowed on the system. The following message is received:
  108. ORA-00257 archiver is stuck. Connect internal only, until freed

    What is the problem

    The archive destination is probably full, backup the archive logs and remove them and the archiver will re-start.

     

  109. Where would you look to find out if a redo log was corrupted assuming you are using Oracle mirrored redo logs
  110. There is no message that comes to the SQLDBA or SRVMGR programs during startup in this situation, you must check the alert.log file for this information.

     

  111. You attempt to add a datafile and get:
  112. ORA-01118: cannot add anymore datafiles: limit of 40 exceeded

    What is the problem and how can you fix it

    When the database was created the db_files parameter in the initialization file was set to 40. You can shutdown and reset this to a higher value, up to the value of MAX_DATAFILES as specified at database creation. If the MAX_DATAFILES is set to low, you will have to rebuild the control file to increase it before proceeding.

     

  113. You look at your fragmentation report and see that smon hasn?t coalesced any of you tablespaces, even though you know several have large chunks of contiguous free extents. What is the problem
  114. Check the dba_tablespaces view for the value of pct_increase for the tablespaces. If pct_increase is zero, smon will not coalesce their free space.

     

  115. Your users get the following error:
  116. ORA-00055 maximum number of DML locks exceeded

    What is the problem and how do you fix it

    The number of DML Locks is set by the initialization parameter DML_LOCKS. If this value is set to low (which it is by default) you will get this error. Increase the value of DML_LOCKS. If you are sure that this is just a temporary problem, you can have them wait and then try again later and the error should clear.

     

  117. You get a call from you backup DBA while you are on vacation. He has corrupted all of the control files while playing with the ALTER DATABASE BACKUP CONTROLFILE command. What do you do

As long as all datafiles are safe and he was successful with the BACKUP controlfile command you can do the following:

CONNECT INTERNAL

STARTUP MOUNT

(Take any read-only tablespaces offline before next step ALTER DATABASE DATAFILE .... OFFLINE;)

RECOVER DATABASE USING BACKUP CONTROLFILE

ALTER DATABASE OPEN RESETLOGS;

(bring read-only tablespaces back online)

Shutdown and backup the system, then restart

If they have a recent output file from the ALTER DATABASE BACKUP CONTROL FILE TO TRACE; command, they can use that to recover as well.

If no backup of the control file is available then the following will be required:

CONNECT INTERNAL

STARTUP NOMOUNT

CREATE CONTROL FILE .....;

However, they will need to know all of the datafiles, logfiles, and settings for MAXLOGFILES, MAXLOGMEMBERS, MAXLOGHISTORY, MAXDATAFILES for the database to use the command.

  1. What is a CO-RELATED SUBQUERY
    A CO-RELATED SUBQUERY is one that has a correlation name as table or view designator in the FROM clause of the outer query and the same correlation name as a qualifier of a search condition in the WHERE clause of the subquery.
  2. eg
  3. SELECT field1 from table1 X
  4. WHERE field2>(select avg(field2) from table1 Y
  5. where
  6. field1=X.field1);



    (The subquery in a correlated subquery is revaluated for every row of the table or view named in the outer query.)

  7. What are various joins used while writing SUBQUERIES
    Self join-Its a join foreign key of a table references the same table.

    Outer Join--Its a join condition used where One can query all the rows of one of the tables in the join condition even though they don't satisfy the join condition.

    Equi-join--Its a join condition that retrieves rows from one or more tables in which one or more columns in one table are equal to one or more columns in the second table.
  8. What are various constraints used in SQL
    NULL
    NOT NULL
    CHECK
    DEFAULT
  9. What are different Oracle database objects
    TABLES
    VIEWS
    INDEXES
    SYNONYMS
    SEQUENCES
    TABLESPACES etc
  10. What is difference between Rename and Alias
    Rename is a permanent name given to a table or column whereas Alias is a temporary name given to a table or column which do not exist once the SQL statement is executed.
  11. What is a view
    A view is stored procedure based on one or more tables, its a virtual table.
  12. What are various privileges that a user can grant to another user
    SELECT
    CONNECT
    RESOURCE
  13. What is difference between UNIQUE and PRIMARY KEY constraints
    A table can have only one PRIMARY KEY whereas there can be any number of UNIQUE keys. The columns that compose PK are automatically define NOT NULL, whereas a column that compose a UNIQUE is not automatically defined to be mandatory must also specify the column is NOT NULL.
  14. Can a primary key contain more than one columns
    Yes
  15. How you will avoid duplicating records in a query
    By using DISTINCT
  16. What is difference between SQL and SQL*PLUS
    SQL*PLUS is a command line tool where as SQL and PL/SQL language interface and reporting tool. Its a command line tool that allows user to type SQL commands to be executed directly against an Oracle database. SQL is a language used to query the relational database(DML,DCL,DDL). SQL*PLUS commands are used to format query result, Set options, Edit SQL commands and PL/SQL.
  17. Which datatype is used for storing graphics and images
    LONG RAW data type is used for storing BLOB's (binary large objects).
  18. How will you delete duplicating rows from a base table
    DELETE FROM table_name A WHERE rowid>(SELECT min(rowid) from table_name B where B.table_no=A.table_no);

    CREATE TABLE new_table AS SELECT DISTINCT * FROM old_table;

    DROP old_table RENAME new_table TO old_table DELETE FROM table_name A WHERE rowid NOT IN (SELECT MAX(ROWID) FROM table_name GROUP BY column_name)
  19. What is difference between SUBSTR and INSTR
    SUBSTR returns a specified portion of a string eg SUBSTR('BCDEF',4) output BCDE INSTR provides character position in which a pattern is found in a string.

    eg INSTR('ABC-DC-F','-',2) output 7 (2nd occurence of '-')
  20. There is a string '120000 12 0 .125' ,how you will find the position of the decimal place
    INSTR('120000 12 0 .125',1,'.') output 13
  21. There is a '%' sign in one field of a column. What will be the query to find it.
    '\' Should be used before '%'.
  22. When you use WHERE clause and when you use HAVING clause
    HAVING clause is used when you want to specify a condition for a group function and it is written after GROUP BY clause The WHERE clause is used when you want to specify a condition for columns, single row functions except group functions and it is written before GROUP BY clause if it is used.
  23. Which is more faster - IN or EXISTS
    EXISTS is more faster than IN because EXISTS returns a Boolean value whereas IN returns a value.
  24. What is a OUTER JOIN
    Outer Join--Its a join condition used where you can query all the rows of one of the tables in the join condition even though they dont satisfy the join condition.
  25. How you will avoid your query from using indexes
    SELECT * FROM emp Where emp_no+' '=12345;

    i.e you have to concatenate the column name with space within codes in the where condition.

    SELECT /*+ FULL(a) */ ename, emp_no from emp where emp_no=1234;

    i.e using HINTS
  26. What is a pseudo column. Give some examples
    It is a column that is not an actual column in the table.

    eg USER, UID, SYSDATE, ROWNUM, ROWID, NULL, AND LEVEL.

    Suppose customer table is there having different columns like customer no, payments.What will be the query to select top three max payments.

    SELECT customer_no, payments from customer C1 WHERE 3<=(SELECT COUNT(*) from customer C2 WHERE C1.payment <= C2.payment)
  27. What is the purpose of a cluster.
    Oracle does not allow a user to specifically locate tables, since that is a part of the function of the RDBMS. However, for the purpose of increasing performance, oracle allows a developer to create a CLUSTER. A CLUSTER provides a means for storing data from different tables together for faster retrieval than if the table placement were left to the RDBMS.
  28. What is a cursor.
    Oracle uses work area to execute SQL statements and store processing information PL/SQL construct called a cursor lets you name a work area and access its stored information A cursor is a mechanism used to fetch more than one row in a Pl/SQl block.
  29. Difference between an implicit & an explicit cursor.
    PL/SQL declares a cursor implicitly for all SQL data manipulation statements, including quries that return only one row. However,queries that return more than one row you must declare an explicit cursor or use a cursor FOR loop.

    Explicit cursor is a cursor in which the cursor name is explicitly assigned to a SELECT statement via the CURSOR...IS statement. An implicit cursor is used for all SQL statements Declare, Open, Fetch, Close. An explicit cursors are used to process multirow SELECT statements An implicit cursor is used to process INSERT, UPDATE, DELETE and single row SELECT. .INTO statements.
  30. What are cursor attributes
    %ROWCOUNT
    %NOTFOUND
    %FOUND
    %ISOPEN
  31. What is a cursor for loop.
    Cursor For Loop is a loop where oracle implicitly declares a loop variable, the loop index that of the same record type as the cursor's record.
  32. Difference between NO DATA FOUND and %NOTFOUND
    NO DATA FOUND is an exception raised only for the SELECT....INTO statements when the where clause of the querydoes not match any rows. When the where clause of the explicit cursor does not match any rows the %NOTFOUND attribute is set to TRUE instead.
  33. What a SELECT FOR UPDATE cursor represent.
    SELECT......FROM......FOR......UPDATE[OF column-reference][NOWAIT] The processing done in a fetch loop modifies the rows that have been retrieved by the cursor.
    A convenient way of modifying the rows is done by a method with two parts: the FOR UPDATE clause in the cursor declaration, WHERE CURRENT OF CLAUSE in an UPDATE or declaration statement.
  34. What 'WHERE CURRENT OF ' clause does in a cursor.
  35. LOOP
  36. SELECT num_credits INTO v_numcredits FROM classes
  37. WHERE dept=123 and course=101;
  38. UPDATE students
  39. SET current_credits=current_credits+v_numcredits
  40. WHERE CURRENT OF X;
  41. END LOOP
  42. COMMIT;
  43. END;

     

  44. What is use of a cursor variable? How it is defined.
    A cursor variable is associated with different statements at run time, which can hold different values at run time. Static cursors can only be associated with one run time query. A cursor variable is reference type(like a pointer in C).
    Declaring a cursor variable: TYPE type_name IS REF CURSOR RETURN return_type type_name is the name of the reference type,return_type is a record type indicating the types of the select list that will eventually be returned by the cursor variable.
  45. What should be the return type for a cursor variable.Can we use a scalar data type as return type.
    The return type for a cursor must be a record type.It can be declared explicitly as a user-defined or %ROWTYPE can be used. eg TYPE t_studentsref IS REF CURSOR RETURN students%ROWTYPE
  46. How you open and close a cursor variable.Why it is required.
    OPEN cursor variable FOR SELECT...Statement CLOSE cursor variable In order to associate a cursor variable with a particular SELECT statement OPEN syntax is used.In order to free the resources used for the query CLOSE statement is used.
  47. How you were passing cursor variables in PL/SQL 2.2.
    In PL/SQL 2.2 cursor variables cannot be declared in a package.This is because the storage for a cursor variable has to be allocated using Pro*C or OCI with version 2.2,the only means of passing a cursor variable to a PL/SQL block is via bind variable or a procedure parameter.
  48. Can cursor variables be stored in PL/SQL tables.If yes how.If not why.
    No, a cursor variable points a row which cannot be stored in a two-dimensional PL/SQL table.
  49. Difference between procedure and function.
    Functions are named PL/SQL blocks that return a value and can be called with arguments procedure a named block that can be called with parameter. A procedure all is a PL/SQL statement by itself, while a Function call is called as part of an expression.
  50. What are different modes of parameters used in functions and procedures.
    IN
    OUT
    INOUT
  51. What is difference between a formal and an actual parameter
    The variables declared in the procedure and which are passed, as arguments are called actual, the parameters in the procedure declaration. Actual parameters contain the values that are passed to a procedure and receive results. Formal parameters are the placeholders for the values of actual parameters
  52. Can the default values be assigned to actual parameters.
    Yes
  53. Can a function take OUT parameters.If not why.
    No.A function has to return a value,an OUT parameter cannot return a value.
  54. What is syntax for dropping a procedure and a function .Are these operations possible.
  55. Drop Procedure procedure_name
  56. Drop Function function_name
  57.  

  58. What are ORACLE PRECOMPILERS.
    Using ORACLE PRECOMPILERS ,SQL statements and PL/SQL blocks can be contained inside 3GL programs written in C,C++,COBOL,PASCAL, FORTRAN,PL/1 AND ADA.
    The Precompilers are known as Pro*C,Pro*Cobol,... This form of PL/SQL is known as embedded pl/sql,the language in which pl/sql is embedded is known as the host language.
    The prcompiler translates the embedded SQL and pl/sql ststements into calls to the precompiler runtime library.The output must be compiled and linked with this library to creater an executable.
  59. What is OCI. What are its uses.
    Oracle Call Interface is a method of accesing database from a 3GL program. Uses--No precompiler is required,PL/SQL blocks are executed like other DML statements.
  60. The OCI library provides
  61. -functions to parse SQL statemets
  62. -bind input variables
  63. -bind output variables
  64. -execute statements
  65. -fetch the results
  66.  

  67. Difference between database triggers and form triggers.
    a) Data base trigger(DBT) fires when a DML operation is performed on a data base table.Form trigger(FT) Fires when user presses a key or navigates between fields on the screen
    b) Can be row level or statement level No distinction between row level and statement level.
    c) Can manipulate data stored in Oracle tables via SQL Can manipulate data in Oracle tables as well as variables in forms.
    d) Can be fired from any session executing the triggering DML statements. Can be fired only from the form that define the trigger.
    e) Can cause other database triggers to fire.Can cause other database triggers to fire,but not other form triggers.
  68. What is an UTL_FILE.What are different procedures and functions associated
    with it. UTL_FILE is a package that adds the ability to read and write to operating system files Procedures associated with it are FCLOSE, FCLOSE_ALL and 5 procedures to output data to a file PUT, PUT_LINE, NEW_LINE, PUTF, FFLUSH.PUT, FFLUSH.PUT_LINE,FFLUSH.NEW_LINE. Functions associated with it are FOPEN, ISOPEN.
  69. Can you use a commit statement within a database trigger.
    No
  70. What is the maximum buffer size that can be specified using the DBMS_OUTPUT.ENABLE function?
    1,000,000

Oracle Interview Questions

  1. How would you determine the time zone under which a database was operating?
  2. Explain the use of setting GLOBAL_NAMES equal to TRUE.
  3. What command would you use to encrypt a PL/SQL application?
  4. Explain the difference between a FUNCTION, PROCEDURE and PACKAGE.
  5. Explain the use of table functions.
  6. Name three advisory statistics you can collect.
  7. Where in the Oracle directory tree structure are audit traces placed?
  8. Explain materialized views and how they are used.
  9. When a user process fails, what background process cleans up after it?
  10. What background process refreshes materialized views?
  11. How would you determine what sessions are connected and what resources they are waiting for?
  12. Describe what redo logs are.
  13. How would you force a log switch?
  14. Give two methods you could use to determine what DDL changes have been made.
  15. What does coalescing a tablespace do?
  16. What is the difference between a TEMPORARY tablespace and a PERMANENT tablespace?
  17. Name a tablespace automatically created when you create a database.
  18. When creating a user, what permissions must you grant to allow them to connect to the database?
  19. How do you add a data file to a tablespace?
  20. How do you resize a data file?
  21. What view would you use to look at the size of a data file?
  22. What view would you use to determine free space in a tablespace?
  23. How would you determine who has added a row to a table?
  24. How can you rebuild an index?
  25. Explain what partitioning is and what its benefit is.
  26. You have just compiled a PL/SQL package but got errors, how would you view the errors?
  27. How can you gather statistics on a table?
  28. How can you enable a trace for a session?
  29. What is the difference between the SQL*Loader and IMPORT utilities?
  30. Name two files used for network connection to a database.

Sources :
DEVFYI - Developer Resource - FYI
TechGuider


Click here to get Interview's Topic Index